Вы находитесь на странице: 1из 60

P hysi cs | 21.

35

Solved Examples

JEE Main/Boards 10-9C and moving in the x-y plane. Find the velocity of
the particle.
Example 1: A uniform magnetic fields of 30 mT
Sol: The force on the particle in external magnetic
exists in the +X direction. A particle of charge +e and   
mass1.67 x 10-27 kg is projected into the field along the field is F = q(v xB) . Take vector product of velocity and
+Y direction with a speed of 4.8 x 106 m/s magnetic field vector.

( )
(i) Find the force on the charged particle in magnitude 
= Given, B = 4 x10−3 k T,q 10−9 C
and direction

(ii) Find the force if the particle were negatively charged. and Magnetic force = Fm (4.0i + 3.0 j)10−10 N

(iii) Describe the nature of path followed by the particle Let Velocity of the particle in x-y plane be, v = ν x i + ν y j
  
in both the cases. Then From the relation, Fm = q(v xB)
Sol: The force on the particle in external magnetic We have,

( )( )
  
field is F = q(v xB) . Take vector product of velocity and (4.0i + 3.0j) x10 10−9  ν x i + ν y j x 4x10−3 k 
−10
=
 
magnetic field vector, and solve for force.
y (
= 4ν X10−12 j − 4ν 10−12 j
y x )
Comparing the coefficients of i and j we have,

v

F 4 x 10-10= 4ν y x10−12
∴ =ν y 102=
m / s 100m / s

x and 3.0 x 10-10= 4ν y x10−12


 
B ∴ ν x =−75m / s ; ∴ V = −75i + 100j

z
Example 3: Figure shows current loop having two
(i) Force acting on a charge particle moving in the circular arcs joined by two radial lines. Find the magnetic
magnetic field field B at the center O.
   
F = q(v xB) Magnetic field B = 30(mT)j i

Velocity of the charge particle V = 4.8 x106 (m / s) j D C

( )( )( )

F = 1.6 x10−19  4.8 x106 j x 30 x10−3 i  A B
 

F =230.4 x 10-16( −k̂ )N.
O
(ii) If the particle were negatively charged, the magnitude
Sol: Find magnetic field at the center O of concentric
of the force will be the same but the direction will be
µ Iθ
along (+z) direction. arcs AB and CD by B = 0 where θ is the angle
4 πR
(iii) As v ⊥ B, the path describe is a circle subtended at the center.
mv
R
= = (1.67x10−27 ) ⋅ (4.8x106 ) / Magnetic field at point O, due to wires CB and AD
qB
(1.6x10−19 ) ⋅ (30x10−3 ) = 1.67 m. will be zero. Magnetic field due to wire BA will be,
 θ  µ i  
B1 =    0  Direction of field B1 is coming out of
Example 2: A magnetic field of (4.0 x 10-3 k ) T exerts a  2π   2a 
force (4.0 i +3.0 j ) x 10-10N on a particle having a charge the plane of the figure. Similarly, field at O due to arc
2 1 . 3 6 | Moving Charges and Magnetism

of uniform magnetic field only, the particle performs


 θ  µ i 
DC will be, B2 =    0  mv
 2π   2a  uniform circular motion of radius r = .
 qB
Direction of field B2 is going into the plane of the Let i , j and k be unit vector along the positive directions
figure. The resultant field at O is of x, y and z axes. Q=charge on the particle=1.6 x 10-19C,
v=velocity of the charged particle
µ0 iθ(b − a)
B = B1 − B2 = Coming out of theplane, z
4 π ab

Example 4: A current of 2.00 A exist in a square loop of


edge 10.0 cm. Find the magnetic field B at the center of
the square loop.

Sol: The center of the loop is equidistant from all x


the sides, and can be considered as a point on the
perpendicular bisector of one side.The field at the point
y
due to one side is
µo Ia =(1.28 x 106)ms-1
B= 
2πd a2 + 4 d2 E = electric field intensity;

=(-102.4 x 103Vm-1) k B =magnetic induction of the
The magnetic field at the center due to the four sides
magnetic field = (8 x 10-2 Wbm-2) j
will be equal in magnitude and direction. The field due 
to one side will be ∴ Fe =electric force on the charge

µ0ia =qE=1.6 x 10-19(-102.4x103)N k = 163.84 x 10-16N(- k )


B1 =
2πd a2 + 4 d2 Fm =magnetic force on the charge = qv x B
= [1.6x10-19(1.28x106)(8x10-2)N]( i x j )= (163.84x10-16N)
Here, a=10 cm and d=a/2=5 cm.
( k )
 
µ0 (2 A)  10 cm

The two forces Fe and Fm are along z-axis and equal,
Thus, B1 =  
2π(5cm)  (10 cm)2 + 4 (5cm)2  opposite and collinear. The net force on the charge is
 
zero and hence the particle does not get deflection and
1 continues to travel along x-axis. (a) At time t=5x10-6s
=2 x 10-7 T mA-1 x 2 A x = 5·66 x 10-6 T
5 2 cm
x=(5 x 10-6)(1.28 x 106)=6.4m∴ Coordinates of the
Hence, the net field at the center of the loop will be particle = (6.4 m,0,0)
4 x 5·66 x 10-6T=22·6x10-6 T. (b) When the electric field is switched off, the particle
is in the uniform magnetic field perpendicular to its
velocity only and has a uniform circular motion in the
Example 5: A particle of mass 1 x 10-26kg and charge
x-z plane (i.e. the plane of velocity and magnetic force),
1.6 x 10-19C travelling with a velocity 1.28 x 106 ms-1
anticlockwise as seen along+ y axis.
in the +x direction enters a region in which uniform
magnetic field of induction B are present such that mv 2
Now, =qvB where r is the radius of the circle.
Ex=Ey=0, Ez = -102.4 kVm-1 and Bx = Bz=0. By=8 x 10-2. r
The particle enters this region at the origin at time
∴r =
mv
=
( )(
1x10−26 1.28 x106 )=1
t = 0. Determine the location (x, y and z coordinates)
of the particle at t= 5 x 10-6s. If the electric field is
qB ( )(
1.6x10−19 8x10−2 )
switched off at this instant (with the magnetic field still
The length of the arc traced by the particle in [(7.5-5)
present), what will be the position of the particle at
x 10-6s]
t = 7.45 x 10-6 s?
1
= (v)(T)=(1.28 x 1060)(2.45 x 10-6)=3.136m= π m=
Sol: In presence of simultaneous electric and magnetic circumference 2
   
( )
field, the Lorentz force is F= q E + (v× B) . Under action ∴ The particle has the coordinates (6,4,0,2m) as (x,y,z).
P hysi cs | 21.37


Example 6: The region between x=0 and x=L is filled Example 7: A uniform, constantmagnetic field B is
directed at an angle of 45° to the x-axis in the xy-plane.
with uniform, steady magnetic field B0k. A particle of
PQRS is a rigid, square wire frame carrying a steady
mass m, positive charge q and velocity v0 î travels along current I0 , with its center at the origin. O. At time t=0,
x-axis and enters the region of magnetic field. Neglect the frame is at rest in the position (shown the Figure)
gravity throughout the question. with its sides parallel to the x and y axes. Each side of
(i) Find the value of L if it emerges from the region of the frame is of mass M and length L.
magnetic field with its final velocity at an angle 30o to y
the initial velocity. S R
(i) Find the final velocity of the particle and the time
spent by it in the magnetic field, if the field now extents
up to x=2. 1L. x
O
Sol: The particle under action of uniform magnetic field
performs uniform circular motion. The magnetic force P Q
acting on it provides the centripetal force. The radius of
mv
the circular orbit is r = .
qB (a) What is the torque τ about O acting on the frame
(i) As the initial velocity of the particle is perpendicular due to the magnetic field?
to the field the particle will move along the arc of a (b) Find the angle by which the frame rotates under the
circle as shown. action of this torque in a short interval of time ∆t , and
v0 i x=L the axis about which this rotation occurs. ( ∆t is so short
o
30 that any variation i n the torque during this interval may
be neglected). Given moment of any variation in the
torque during this interval may be neglected). Given
r B0k moment of inertia of the frame about an axis through
r its center perpendicular to its p late is (4/3) ML2 .
o
30
  
Sol: The torque acting on loop is τ = M × B .

If r is the radius of the circle, then =


α
dω τ
= and
dt I
mv 20
= qv 0B0 Also from geometry, L=r sin 30° y
r
S R
mv 0
⇒r =2L or L =
2qB0  o
45 x
2.1mv 0
(ii) In this =
case L > r Hence the particle will O
2qB0
I0
complete a semi-circular path and emerge from the P Q
field with velocity v 0 ˆi as shown. Time spent by the
πr πm θ = ∫ ωdt (a) As magnetic field B is in x-y plane and
particle in the magnetic field =T = subtends an angle of 45° with x-axis.
v 0 qB0
=Bx Bcos
= 45° B 2
L>r
and
= B y Bsin
= 45° B 2

So in vector from

( ) ( )

=B ˆi B 2 + j B 2
v0 i 
and=
M I= ˆ I L2kˆ
0 Sk 0
The speed of the particle does not change due to the
magnetic field.
2 1 . 3 8 | Moving Charges and Magnetism

   B ˆ B ˆ Consider the figure.


so, τˆ= M × B= I0L2kˆ ×  i+ j
y
 2 2 
I0L2B
i.e.,
= τˆ
2
(
× −ˆi + ˆj ) 2
i.e., torque has magnitude I0L2B and is directed along
line QS from Q to S. 4
1
(b) As by theorem of perpendicular axes, moment of
inertia of the frame about QS, 3 P
x
1 14 2 2 2 a/2
I=
QS = I  ML= ML 3a/2
2 z 2 3  3 5
And as τ = Iα , z
    
( ) + (B ) + (B ) + (B ) + (B )
2 
τ I L B × 3 3 I0B BP = B1 2 3 4 5
α= = 0 = P P P P P
1 2L2M 2 M
 µ0i
As here α is constant, equations of circular motion are
where =
B1 ( ) P  3a 
( −ˆj)
valid and hence from 4π  
 2
1  µ0 i 
θ = ω0 t + αt2 with ω0 =0 we have
2 (Semi-infinite wire) =B2 ( ) P  3a 
( +kˆ ) (B ) 3
P
=0 ;
4 
1 2 1  3 I0B  3 I0B 2  2
 ( ∆t ) =
2  µ0 i
θ=
2
αt = 
22 M  4 M
∆t
( )
B4
=
P a
( −kˆ )
4 
2
Example 8: In the figure shown the magnetic field at
the point P.  µ i   1 1 ˆ  1 
⇒ BP = 0  −  +  j −  1 −  kˆ 
y 2a   3π π   3 

 2µ0i  1 ˆ ˆ   µ0i
BP
⇒=  j − k ⇒ BP
= 1 + π2
3a  π  3πa

Example 9: What is the smallest value of B that can


i be set up at the equator to permit a portion of speed
P 107 m s to circulate around the earth?
x
(a, 0) (2a, 0) (3a, 0) R = 1.67 × 10−27 kg .
6.4 × 106 m,mp =
 
i
Sol: Particle under action of force in uniform magnetic
z
field, moves in circular orbit whose radius is given by
Sol: The conductor forms two concentric semicircles mv
r= . For charged particle orbiting near earth with
Bq
and two straight wires. Find magnetic field at the center high velocity, the magnetic field can be obtained
µ0 Iθ rearranging above formula.
P due to concentric arcs by formula B = , and fields
4 πR mv
From the relation r =
µ0 I Bq
due to straight wires by formula B = and then add mv
4 πd We have B =
the fields due to individual parts. qr
Substituting the values,we have
P hysi cs | 21.39

B
=
(1.67 × 10 )(10 )=
−27 7

1.6 × 108 T f = q  V × B 


f = q V × B (( ))
µ I
((
= q −V0 i ×  4R − i + j 

))
= q −V0 ˆˆi ×  µ00 I −ˆˆi + ˆˆj 
(1.6 × 10 )(6.4 × 10 )
−19 6

 qV µ I 
 4R

qV µ I


 qV µ I  ( ) (−ˆˆi × −ˆˆi= ))
+ ˆˆj  qV0 µ0 I −kˆˆ(( ))
 4R  ( ) (
0 0 0 0
= 0 0
=  4R   − i × − i= +j  4R −k
 4R
 

JEE Advanced/Boards Example 2: A long horizontal wire AB, which is free to


move in a vertical plane and carries a steady current of
20 A, is in equilibrium at a height of 0.01 m over another
parallel long wire CD which is fixed in a horizontal plane
Example 1: A circular loop of radius R is bent along a
and carries a steady current of 30 A, as shown in figure
diameter and given a shape as shown in figure. One of
Show that when AB is slightlydeed it executes simple
the semi-circle (KNM) lies in the x-z plane and the other
harmonic motion. Find the period of oscillation.
one (KLM) in the y-z plane with their centers at origin.
Current I is flowing through each of the semi-circles as A B
shown in Figure.
Sol: The current carrying wire AB, experiences force
L due to the magnetic field created by wire CD. Find the
equation of motion of wire AB. If the force acting on
wire AB is restoring in nature and directly proportional
M y

i to its displacement from the equilibrium position,


then we compare the equation of acceleration with
N
the standard differential equation of SHM. Then time
x
i
ω
period of oscillation is given by T = 2π
K z
Let m be the
g
A particle of charge q is released at the origin with a
mass per unit length of wire AB. At a height x about the
velocity V = −V0 ˆi . Find the instantaneous force F in the
particle. Assume that space is gravity free. wire AB will be given by
Fm
Sol: For wire bent as shown the magnetic field at the i1=20A
µ I( π) A B
center is calculated as B = 0 , where π is the angle
4 πR Fg
X=d=0.01
subtended by the wire at center. The Lorentz force
   C i2=30A D
acting on particle is =
F q(v× B)
µ0i1i2
Magnetic field at the center of a circular wire of radius Fm = (upwards)  … (i)
µ I 2πx
R carrying a current I is given by B = 0
2R Wt. per unit of wire AB is Fg =mg (downwards) At x=d,
wire in equilibrium
In this problem, current are flowing in two semi-circles,
KLM in the y-z plane and KNM in the x-z plane. The µ0 i1i2
i.e., Fm =
Fg ⇒ mg
=
centers of these semi-circles coincide with the origin of 2π d
the Cartesian system of axes. µ0i1i2
mg
  ⇒ =  …(ii)
1  µ0 I  ˆ 1  µ0 I  ˆ d
( ) ( )
2
∴ BKLM 2πd
=   − i ∴ BKNM
=   −j
2  2R  2  2R 
When AB is deed, x decreases therefore, Fm will increase,
µ0 I Fg remains the same. Let
The total magnetic field at the origin is B=
0
4R
( −ˆi + ˆj)
AB is displaced by dx downwards.
It is given that a particle of charge q is released at the Differentiating equation (i) w.r. t.x, we get
origin with a velocity V = −V0 ˆi . The instantaneous force µ0 i1i2
dFm = − dx  …(iii)
acting on this particle is given by 2π x 2
i.e., restoring force, F=d Fm ∝ −dx
2 1 . 4 0 | Moving Charges and Magnetism

Hence the motion of wire is simple harmonic. From


µ0 I2 L xdx µ0 I2 a2 + L2
equation (ii) and (iii), we can write
π ∫0 a2 + x2
= ⇒F = ln
2π a2
 mg 
dFm = −   .dx (x=d)
 d  If the current in B is reversed, the magnetic field due to
g the two wires would be only along
∴Acceleration of wire, a=-   .dx
d Y
Hence period of oscillations B
R
dx disp. a
T=
2π 2π
= P(x,0,0)
a acc. O x
X

0.01 
⇒ T =2π d g =2π ⇒ T =0.2s a  
9.8 R BB BA
Z A
Example 3: A straight segment OC (of length L meter)
of a circuit carrying a current 1 amp is placed along the
x-axis. Two infinitely long straight wire A and B, each x- direction and the force on the current along
extending z = −∞ to + ∞ are fixed at y=-a meter and
x- direction will be zero.
y=+a meter respectively, as shown in the figure. If the
wires A and B each carry a current 1 amp into the plane
of the paper, obtain the expression for the force acting Example 4: Two long wires a and b, carrying equal
on segment OC. What will be the force on OC if the currents of 10.0 A, are placed parallel to each other
current in the wire B is reversed? with a separation of 4.00 cm between them as shown in
figure. Find the magnetic field B at each of the points
Y P, Q and R.
(0,a,0)
B 2.00 cm 2.00 cm 2.00 cm 2.00 cm
i i C
X P a Q b R
O

A Sol: Net field at a point will be the vector sum of the


Z (0,-a,0) fields due to the two wires.
i
The magnetic field at P due to the wire a has magnitude
Sol: Find the net field due to wires A and B at any point
on the wire OC.Find the force due to this field on a small µ0i 4 π × 10−7 TmA −1 × 10A
current element of wire OC at that point. Then integrate B1
= = = 1.00 × 10−4 T.
2πd 2π × 2 × 10−2 m
this expression to find force on wire OC.
Its direction will be perpendicular to the line shown and
Magnetic field BA produced at P(x,0, 0) due to wire, will point downward in the figure. The field at this point
BA =µ0 I 2πR, BB =µ0 I 2πR . due to the other wire has magnitude
Components of BA and BB along x-axis cancel, while
µ0i 4 π × 10−7 TmA −1 × 10A
those along y-axis add up to give total field. B2
= = = 0.33 × 10−4 T.
2πd −2
2π × 6 × 10 m
µ I 2µ0 I x µ0 I x
=B 2  0 = cos θ =
 2πR  2πR R π a + x2
2
( ) Its direction will be the same as that of B1 . Thus, the
resultant field will be 1.33 × 10−4 T also along the same
(along – y direction) direction.

The force dF acting on the current element is Similarly, the resultant magnetic field at R will be
= 1.33 × 10−4 T along the direction pointing upward in
dF = I(d xB) the figure.
µ0 I2x dx The magnetic field at point Q due to the two wires will
=dF = ∴ sin90° 1
π a2 + x2  have equal magnitudes but opposite directions and
hence the resultant field will be zero.
P hysi cs | 21.41

Example 5: A coil of radius R carries current I. Another According to law of conservation of angular momentum,
concentric coil of radius (r<<R) carries current i. Planes I1 ω1 = I2 ω2
of two coils are mutually perpendicular and both the
and according to law of conservation of energy,
coils are free to rotate about a common diameter. Find
maximum kinetic energy of smaller coil when both 1 1
I ω2 + I ω2 =U
the coils are released, masses of coils are M and m 2 1 1 2 2 2
respectively. From above equations, maximum kinetic energy of
1 smaller coil,
Sol: For rotating coils, kinetic energy is I ω2 .
2 1 UI1 µ0 πliMRr 2
2
I=
ω =
Each coil is a magnetic dipole and has a potential
energy in magnetic field due to other coil. This potential
(
2 2 2 I1 + I2 2 MR 2 + mr 2 )
energy is converted into kinetic energy as the dipole
moment of the coil aligns itself with the magnetic field. Example 6: A wire loop carrying a current I is placed in
the x-y plane as shown in Figure.
(a) If a particle with charge q and mass m is placed at
the centerP and given a velocity v along NP find its
instantaneous acceleration.
y
M
If a magnetic dipole having moment M be rotated
through angle θ from equilibrium position in a uniform
magnetic field B, work done on it is W= MB(1 − cos θ) .
q x
This work is stored in the system in the form of energy. 120
o

i
When system is release, dipole starts to rotate to P
occupy equilibrium position and the energy converts
a
into kinetic energy and kinetic energy of the system is
maximum when stored energy is completely released.
N

Magnetic induction, at centers due to current in larger


µ i (b) If an external uniform magnetic induction B = Biˆ is
coil B = 0 Magnetic dipole moment of smaller applied, find the force and torque acting on the loop.
2R
coil is iπr 2 . Initially planes of two coils are mutually
Sol: Find the net magnetic field at the point P due to the
perpendicular, therefore θ is 90° or energy of the arc and the straight wire and find the magnetic force on
q by rules of vector cross product. The magnetic force
system is U = ( )
iπr 2 B (1 − cos90° ) on a current loop in uniform magnetic field is zero.
µ0 Iiπr 2 The toque will be non-zero depending on the angle
U= between field and the area vector of the loop.
2R
When coils are released, both the coils start to rotate (a) As in case of current-carrying straight conductor
about their common diameter and their kinetic energies and arc, the magnitude of B is given by
are maximum when they become coplanar.
y
Moment of inertia of larger coil about axis of rotation is
M
1 1
I1 = mR 2 and that of smaller coil is I2 = mr 2 .
2 2 o
v
90
Since, two coils rotate due to their mutual interaction
only, therefore, if one coil rotates clockwise then the F q x
other rotates anticlockwise. i o P
120
Let angular velocities of larger and smaller coils be a
numerically equal to ω1 and ω2 respectively when they
become coplanar, N
2 1 . 4 2 | Moving Charges and Magnetism

µ0i µ 0 Iφ Example 7: A disc of radius R rotates at an angular


=B1
4 πd
( sin α + sinβ ) and B2 =
4 πr velocity ω about the axis perpendicular to its surface
and passing through its center. If the disc has a uniform
So in accordance with right hand screw rule, charge density σ , find the magnetic induction on the
 axis of rotation at a
( )
B=W
µ0 1
4 π ( acos60 )
ˆ and due to are
× 2sin60( −k)
Sol: The disc can be thought as made-up of elementary
 rings. When disc rotates about axis passing through
(B ) =
MN
µ0 I  2  ˆ
( )
×  π  +k
4π a  3 
center and perpendicular to plane of disc, then each
elementary ring constitutes a current. The magnetic
 field along axis of rotation due to each elementary ring
and hence net B at P due to the givenloop
is to be considered.
  
B BW + BA
= At distance r from the center of disc consider a ring of
radius r and width dr.
 µ 2I  π ˆ
⇒B
= 0
 3 −  ( −k)  …(i) Charge on the ring, dq =( 2πr dr ) σ
4π a  3
dq ωdq
Now as force on charged particle in a magnetic fields Current due to ring is dI = = = σωr dr
T 2π
is given by
Magnetic field due to ring at point P on axis is
  
= (
F q v ×B ) dB =
µ0 dlr 2
or
( )
 32
So=
here, F qvBsin90° along PF 2 r 2 + x2
µ0 σω R r3dr
 µ 2qvI  π
=B ∫=
dB ∫  …(i)
2
(r )
32
=i.e. F 0
 3 −  along PF 0 2
+x 2
4π a  3

 F 2qvI  π Putting r 2 + x2 =
t2 and 2r dr=2t dt and integrating (i)
a = 10−7
and so =  3 −  along PF we get
m a  3
      µ0 σω  R 2 + 2x2 
(b) As dF = =B  − 2x  .
∫ IdL ×B
IdL × B, so F = 2  R 2 + x2 

As here I and B are constant
Example 8: In the figure a charged sphere of mass m
=F I  ∫ dL =
× B 0 as ∫=
dL 0 
and charge q starts sliding from rest on a vertical fixed
circular track of radius R from the position shown. There
Further as area of coil, exists a uniform and constant horizontal magnetic field
 1 1  of induction B. The maximum force exerted by the track
=  πa2 − .2a sin 60° × a cos 60° kˆ
S on the sphere.
3 2 
π 3ˆ m
= a2  − k q
 3 4 
  π 3ˆ
So M = Ia2  −
= IS k
 3 4  X

B
   π 3 ˆ ˆ
and hence τ= M × B= Ia2B  −  k×i
 3 4 
( )
 π 3ˆ
i.e. τ Ia2B  −
=
3 4
(
 jN − m as kˆ × ˆi =ˆj . )
 
Sol: As the sphere moves along the circular track the
vector sum of radial component of magnetic force, the
P hysi cs | 21.43

normal reaction and the radial component of weight of By symmetry, U2 = −U1


the sphere provide the necessary centripetal force.
⇒ −∆U = work done
Fm = qvB , and directed radially outward.
l2
mv 2 mv 2
∴N − mgsin θ + qvB = ⇒
= N + mgsin θ − qvB
R R

Hence at θ = π 2
r
2mgR l1
⇒ Nmax= + mg − qB 2gR
R

= 3mg − qB 2gR. µ IIl b


− (U2 − U1 ) =
= 2 0 1 2 ln .
2π a
Example 9: What is the work done in transferring the The work done in transferring the wire from
wire from position (1) to position (2)? µ0 I1 I2l b
Position 1 to 2 = ln
Sol: While transfering wire from position 1 to position π a
2 find the change in the potential energy of the loop in
the field of the wire. This chage in potential energy will Example 10: A long, straight wire carries a current i. A
be equal to the work done. particle having a positive charge q and mass m, kept at
a distance x0 from the wire is projected towards it with
The loop can be considered as the combination of the a speed v. Find the minimum separation between the
number of elementary loops. The net current in the wire and the particle.
dotted wires is 0 as current in the neighboring loops
flowing through the same wire opposite in direction. Y
Consider an elementary loop of width dr at a distance
r from the wire
i

N x0
ma
O
2 P
A 2 m/s
A
B X
mg
The 'dµ ' magnetic moment of the elemental loop Sol: At minimum separation the x-component of
velocity of the particle will be zero. Find the acceleration
= I2ldr of the particle due to the magnetic force and solve to
get the expression for velocity and displacement.
The B at that point due to straight wire =
µ0 I1 2πr .
Let the particle be initially at P. Take the wire as the y-axis
b and the foot of perpendicular from P to the wire as the
origin. Take the line OP as the x-axis. We have, OP= X0.
l2 The magnetic field B at any point to the right of the
l1 a l a wire is along the negative z-axis. The magnetic force
on the particle is, therefore, in the x-y plane. As there
is no initial velocity along the z-axis, the motion will be
in the x-y plane. Also, its speed remains unchanged. As
1 2 the magnetic field is not uniform, the particle does not
go along a circle.
µ0 I1       µ i 
dU = −B.dµ = − I ldr(cos π) The force at time t is =F qv ×= B q(iv x + jv y ) ×  − 0 k 
2πr 2
 2πx 
[As dµ is anti-parallel to B.]  µ i  µ i
= jqv x 0 − iqv y 0 .
µ0 I1 I2l b 1 µ0 I1 I2l  a  2πx 2πx
U1
= ∫=
du
2π a r ∫ =dr

ln  
b
2 1 . 4 4 | Moving Charges and Magnetism

Fx µ0 qi µ y µy is a constant. Find the magnetic field at a point P at a


Thus ax = = − = −λ  …(i) distance x from the metal sheet.
m 2πm x x
P
µ0 qi
Where λ = .
2πm x

dv x dv x dx v x dv x
Also,=
ax = = . …(ii)
dt dx dt dx
Sol: Field due to the sheet will be symmetric. Field lines
As v 2x + v 2y =
v2 , will be parallel to the sheet at points near it. Select a
rectangular amperian loop and use Ampere’s Law to
2v x dv x + 2v y dv y =
0 find the field.

giving v x dv x = − v y dv y .  …(iii) Consider two strips A and C of the sheet situated


symmetrically on the two sides of P.The magnetic field
From (i), (ii) and (iii), at P due to the strip A is Ba perpendicular to AP and
that due to the strip C is Bc perpendicular to CP. The
v y dv y λv y dx dv y resultant of these two is parallel to the width AC of the
= or = .
dx x x λ sheet. The field due to the whole sheet will also be in
this direction. Suppose this field has magnitude B.
Initially x = x0 and v y = 0 . At minimum separation
from the wire, v x = 0 so that v y = − v . The field on the opposite side of the sheet at the
same distance will also be B but in opposite direction.
x
dx
−v dv y x v Applying Ampere’s law to the rectangle shown in figure.
Thus ∫ = ∫ or, ln = −
x0
x 0
λ x0 λ
2 πmv Ba
− B P
−v λ µ0qi x
or, x x=
= 0e x0 e P

BC x
C x
B
Example 11: Figure shows a cross section of a large O dl
l
metal sheet carrying an electric current along its (a) (b)
surface. The current in a strip of width dl is Kdl where K
1
2Bl = µ0Kl or, B= µ K Note that it is independent of x.
2 0

JEE Main/Boards

Exercise 1 Q.4 A horizontal overhead power line carries a current


of 90 A in east west direction. What is the magnitude
Q.1 A circular coil of wire consisting of 100 turns, each and direction of the magnetic field due to the current
of radius 8.0 cm carries current of 0.40 A. What is the 1.5 m below the line?
magnitude of the magnetic field B at the center of the
coil? Q.5 What is the magnitude of a magnetic force per unit
length on a wire carrying a current of 8 A and making an
Q.2 A long straight wire carries a current of 35 A. What angle of 30° with the direction of a uniform magnetic
is the magnitude of the field B at a point 20 cm from field of 0.15 T?
the wire?
Q.6 In a chamber, a uniform magnetic field of 6.5
Q.3 A long straight wire in the horizontal plane carrier G(1G= 10−4 T) is maintained. An electron is shot into
of 50 A in north to south direction. Give the magnitude the field with a speed of 4.8 × 106 ms−1 normal to the
and direction of Bat a point 2.5 m east of the wire. field. Explain why the path of the electron is a circle.
Determine the radius of the circular orbit.
P hysi cs | 21.45

Q.12 A circular coil of N turns and radius R carries a


(e =
1.6 × 10 −19
9.1 × 10−31 kg
C, me = ) current I. It is unwound and rewound to make another
coil of radius R/2. Current I remaining the same.
Q.7 (i) A circular coil of 30 turns and radius 8.0 cm Calculate the ratio of the magnetic moments of the
carrying a current of 6.0 A is suspended vertically in a new coil and the original coil.
uniform horizontal magnetic field of magnitude 1.0 T.
The field lines make an angle of 60o with the normal of
Q.13 A circular coil of 20 turns and radius 10 cm is
the coil. Calculate the magnitude of the counter torque
placed in a uniform magnetic field of 0.10 T normal to
that must be applied to prevent the coil from turning.
the plane of the coil. If the current in the coil is 5.0 A,
(ii) Would your answer change, if the circular coil in (a) what is the
were replaced by a planner coil of some irregular shape
(a) Total torque on the coil,
that encloses the same area? (All other particulars are
also unaltered.) (b) Total force on the coil
(c) Average force on each electron is the coil due to the
Q.8 Two concentric circular coils X and Y radii 16 cm magnetic field?
and 10 cm, respectively, lie in the same vertical plane
(The coil is made of copper wire of cross-sectional area
containing the north to south direction. Coil X has 20
10−5 m2 , and the free electron density in copper is
turns and carries a current 16 A; coil Y has 25 turns and
given to be about 1029 m−3 .)
carries a current of 18 A. The sense of the current in
X is anticlockwise, and clockwise in Y, for an observer
looking at the coils facing west. Give the magnitude Q.14 State the Biot-Savart law for the magnetic field
and direction of the net magnetic field due to the coils due to a current-carrying element. Use this law to
at their center. obtain a formula for magnetic field at the center of a
circular loop of radius a carrying a, steady current I.
Q.9 A straight horizontal conducting rod of length 0.45
m and mass 60 g is suspended by two vertical wires at Q.15 Give the formula for the magnetic field produced
its ends. A current of 5.0 A is set up in the rod through by a straight infinitely long current-carrying wire.
the wires. Describe the lines of field B in this case.
(a) What magnetic field should be set up normal to the
conductor in order that the tension in the wire is zero? Q.16 How much is the density B at the center of a long
solenoid?
(b) What will be the total tension in the wires if the
direction of current is reversed keeping the magnetic
field same as before? Q.17 A proton shot at normal to magnetic field describe
a circular path of radius R. If a deuteron ( H ) is to
1
2

Q.10 The wires which connect the battery of an move on the same path, what should be the ratio of the
automobile to its starting motor carry a current of 300 velocity of proton and the velocity of deuteron?
A (for a short time). What is the force per unit length
between its wires if they are 70 cm long and 1.5 cm
Q.18 State the principle of cyclotron.
apart? Is the force attractive of repulsive?

Q.19 A charge q is moving in a region where both the


Q.11 A uniform magnetic field of 1.5 T exists in a
magnetic field B and electric field E are simultaneously
cylindrical region of radius10.0 cm, its direction parallel
present. What is the Lorentz force acting on the charge?
to the axis along east to west. A wire carrying current
of 7.0 A in the north to south direction passes through
this region. What is the magnitude and direction of the Q.20 A charged particle moving in a straight line enters
force on the wire if, a uniform magnetic field at an angle of 45° . What will
be its path?
(a) The wire intersects the axis,
(b) The wire is turned from N-S to northeast-northwest Q.21 A current of 1A is flowing in the sides of an
direction,
equilateral triangle of side 4.5 × 10−2 m. Find the
(c) The wire in the N-S direction is lowered from the axis magnetic field at the centroid of the triangle.
by a distance of 6.0 cm?
2 1 . 4 6 | Moving Charges and Magnetism

A 10
m from the wire B, is zero. Calculate
11
l (i) the magnitude and direction of current in B (ii) the
magnitude of magnetic field induction at S
O
l (ii) the force per unit length of the wire B.
o
60
o 60
r

Exercise 2
B D C

Q.22 The radius of the first electron orbit of a hydrogen


atom is 0.5 Å. The electron moves in this orbit with a
uniform speed of 2.2 × 106 ms−1 . What is the magnetic Q.1 A current 1 ampere is flowing through each of
field produced at the center of the nucleus due to the the bent wires as shown figure. The magnitude and
motion of this electron? direction of magnetic field at O is

Q.23 A solenoid is 2 m long and 3 cm in diameter. It R’


has 5 layers of windings of 1000 turns each and carries O
a current of 5 A. What is the magnetic field at itscenter?
R
Use the standard value of µ0 .
µ0i  1 2  µ0i  1 3 
(A)  +  (B)  + 
Q.24 A proton entersa magnetic field of flux density 2.5 4  R R′  4  R R′ 
T with a velocity of 1.5 × 107 ms−1 at an angle of 30° with µ0i  1 µ0i  1 3 
3 
the field. Find the force on the proton. (C)  +  (D)  + 
8  R 2R ′  8  R R′ 

Q.25Two parallel wires one meter apart carry currents of


Q.2 Net magnetic field at the center of the circle O
1A and 3 respectively in opposite directions. Calculate
due to a current carrying loop as shown in figure is
the force per unit length acting between these wires.
( θ < 180°)
Q.26 A solenoid of length 0.4, and having 400 turns
of wire carries a current of 3 A. A thin coil having 10 i i O
turns of wire and radius 0.01 m carries a current 0.4 A.
Calculate the torque required to hold the coil in the
middle of the solenoid with its axis perpendicularto the
axis of the solenoid. (A) Zero
(B) Perpendicular to paper inwards
Q.27 In a circuit shown in figure a voltmeter reads
(C) Perpendicular to paper outwards
30 V, when it is connected across 400ohm resistance.
Calculate what the same voltmeter will read when (D) Is perpendicular to paper inwards if θ ≤ 90° and
connected across the 300 Ω resistance? perpendicular to paper outwards if 90° ≤ θ < 180°
v
Q.3 A charge particle A of charge q=2C has velocity
v=100 m/s. When it passes through point A and
300 400 has velocity in the direction shown. The strength of
magnetic field at point B due to this moving charge is
(r=2 m). v B
o
60v 30
r
Q.28 Two long straight parallel wires are 2m apart,
perpendicular to the plane of the paper. The wire A A
carries a current of 9.6 ampere directed into the plane
(A) 2.5 µT (B) 5.0 µT
of the paper. The wire B carries a current such that the
magnetic field induction at the point P, at a distance of (C) 2.0 µT (D) None
P hysi cs | 21.47

Q.4 Three rings, each having equal radius R, are placed Q.8 A particle of charge q and mass m starts moving
mutually perpendicular to each other and each having from the origin under the action of an electric field
its center at the origin of co-ordinates system. If current ˆ

= E E= 0 i and B B0 ˆi with velocity v = v 0 ˆj . The speed of
is flowing through each ring then the magnitude of the
magnetic field at the common center is the particle will become 2 v 0 after a time
2mv 0 2Bq
y (A) t = (B) t =
qE mv 0

3Bq 3 mv 0
(C) t = (D) t =
mv 0 qE
x
Q.9 An electron is projected with velocity v 0 in a uniform
electric field E perpendicular to the field. Again it is
z projected with velocity v 0 perpendicular to a uniform
magnetic field B. If r1 is initial radius of curvature just
µ0 I
(A) 3 (B) Zero after entering in the electric field and r2 in initial radius
2R of curvature just after entering in magnetic field then
(C) ( 2 −1 ) µ2RI
0
(D) ( 3− 2 ) µ2RI
0 the ratio r1 r2 is equal to

Bv 2 0 B Ev 0 Bv 0
(A) (B) (C) (D)
Q.5 Two concentric coils X and Y of radii 16 cm and E E B E
10 cm lie in the same vertical plane containing N-S
direction. X has 20 turns and carries 16 A. Y has 25 turns Q.10 A uniform magnetic field B = B0 ˆj exists in a space.
& carries 18 A. X has current in anticlockwise direction A particle of mass m and charge q is projected towards
and Y has current in clockwise direction for an observer, negative x-axis with speed v from the point (d, 0, 0).
looking at the coils facing the west. The magnitude of The maximum value v for which the particle does not
net magnetic field at their common center is hit y-z plane is

(A) 5π × 10−4 T towards west


(B) 13π × 10−4 T towards east 2B0 q B0 q B0 q B0 qd
(A) (B) (C) (D)
dm m 2dm 2m
(C) 13π × 10−4 T towards west
(D) 5π × 10−4 T towards east Q.11 Two protons move parallel to each other, keeping
distance r between them, both moving with same
Q.6 Equal current i is flowing in three infinitely long velocity v. Then the ratio of the electric and magnetic
wires along positive x, y and z directions. The magnetic force of interaction between them is.
field at a point (0, 0, -a) would be: (A) c2/v2 (B) 2c2/v2
µ i µ i
2πa
( )
(A) 0 ˆj − ˆi
2πa
( )
(B) 0 ˆi + ˆj (C) c2/2v2 (D) None

µ0i ˆ ˆ µ0i ˆ ˆ ˆ Q.12 Three ions H+ ,He+ and O +2 having same kinetic
(C)
2πa
i−j ( ) (D)
2πa
(
i + j+k ) energy pass through a region in which there width is a
uniform magnetic field perpendicular to their velocity,
Q.7 An electron is moving along positive x-axis. A then:
uniform electric field exists towards negatively y-axis. (A) H+ will be least deflected.
What should be the direction of magnetic field of
(B) He+ and O +2 will be deflected equally.
suitable magnitude so that net force of electron is
zero. (C) O +2 will be deflected most.
(A) Positive z-axis (B) Negative z-axis (D) all will be deflected equally.
(C) Positive y-axis (D) Negative y-axis
Q.13 An electron having kinetic energy T is moving in
a circular orbit of radius R perpendicular to a uniform
2 1 . 4 8 | Moving Charges and Magnetism

magnetic induction B. If kinetic energy is doubled and Q.18 A block of mass m & charge q is released on a
magnetic induction tripled, the radius will become. long smooth inclined plane magnetic field B is constant,
3R 3 uniform, horizontal and parallel to surface as shown.
(A) (B) R Find the time from start when block loses contact with
2 2
the surface.
2 4 mcos θ mcosec θ
(C) R (D) R (A) (B)
9 3 qB qB
mcot θ
Q.14 A charged particle moves in magnetic field (C) (D) None
 qB
B = 10iˆ with initial velocity u= 5i + 4j .
The path of the particle will be. Q.19 A metal ring of radius r=0.5m with its plane normal
to a uniform magnetic field B of induction 0.2T carries
(A) Straight line (B) Circle a current I=100A. The tension in Newton developed in
(C) Helical (D) None the ring is:
q
m
(
Q.15 A electron experiences a force 4.0iˆ + 3.0ˆj x10−13 N ) B

in a uniform magnetic field when its velocity is


2.5k × 107 ms−1 .When the velocity is redirected and 
becomes ( )
1.5iˆ − 2.0ˆj x107 ms−1 , the magnetic force
(A) 100 (B) 50
of the electron is zero. The magnetic field vector B is :
(C) 25 (D) 10
(A) −0.075iˆ + 0.1ˆj (B) 0.1iˆ + 0.075ˆj
(C) 0.075iˆ + 0.1ˆj + kˆ (D) 0.075iˆ + 0.1ˆj Q.20 In the shown a coil of single turn is wound on a
sphere of radius R and mass m. The plane of the coil is
Q.16 An electron moving with a velocity V1 = 2iˆ m / s parallel to the plane and lies in the equatorial plane of
at a point in a magnetic field experiences a force the sphere. Current in the coil is i. The value of B if the
sphere is in equilibrium is
F1 = −2ˆj N . If the electron is moving with a velocity
V = 2ˆj m / s at the same point, it experiences a force
2 B
F = + 2iˆ N . The force the electron would experience if it
2

were moving with a velocity V3 = 2kˆ m / s at the same
point is
mgcos θ mg
(A) Zero (B) 2kˆ N (A) (B)
πiR πiR
(C) −2kˆ N (D) Information is insufficient
mg tan θ mgsin θ
(C) (D)
Q.17 The direction of magnetic force on the electron as πiR πiR
shown in the diagram is along
Q.21 The magnetic moment of a circular orbit of radius
y ‘r’ carrying a charge ‘q’ and rotating with velocity v is
given by
x qvr qvr
(A) (B)
i 2π 2

(C) qvπr (D) qvπr 2


e
(A) y-axis (B) –y-axis (C) z-axis (D) –z-axis
d d d P hysi
d cs | 21.49
(a) (b)

Previous Years’ Questions


d d
Q.1 Two very long straight parallel wires carry steady x x’ x x’
currents I and –I respectively. The distance between the
wires is d. At a certain instant of time, a point charge q is
at a point equidistant from the two wires  in the plane of d d
the wires. Its instantaneous velocity V is perpendicular
(a) (b)
to this plane. The magnitude of the force due to the
magnetic field acting on the charge at this instant is
 (1998) Q.4 A non-planar loop of conducting wire carrying a
current I is placed as shown in the figure. Each of the
µ0 Iqv µ0 Iqv straight section of the loop is of length 2a. The magnetic
(A) (B)
2πd πd field due tothis loop at the point P(a,0,a) points in the
2µ0 Iqv direction (2001)
(C) (D) Zero z
πd y

Q.2 An infinitely long conductor PQR is bent to form a


right angle as shown in Figure. A current I flows through
PQR. The magnetic field due to this current at the point
M is H1. Now, another infinitely long straight conductor x
QS is connected at Q, so that current is I/2 in QR as well
as in QS, the current in PQ remaining uncharged. The
magnetic field at M is now H2. The ratio H1/H2 is given 1 1
by  (2000)
(A)
2
( −ˆj + kˆ ) (B)
3
( −ˆj + kˆ + ˆi )
1 ˆ ˆ ˆ 1 ˆ ˆ
M (C)
3
(
i + j+k ) (D)
2
(
i +k )
Q.5 A coil having N turns is wound tightly in the form of
90
o a spiral with inner and outer radii a and b respectively.
When a current I passes through the coil, the magnetic
 
P l Q 90o S field at the center is  (2001)
µ0NI 2µ0NI
(A) (B)
R b a
µ0NI b µ0 IN b
(C) log (D) log  
 2(b− a) a 2(b− a) a
(A) 1/2 (B) 1
(C) 2/3 (D) 2 Q.6 Two particles A and B of masses mA and mB
respectively and having the same charge are moving in
a plane. A uniform magnetic field exists perpendicular
Q.3 Two long parallel wire are at a distance 2d apart.
to this plane. The speeds of the particles are VA and
They carry steady equal currents flowing out of the
VB respectively and the trajectories are as shown in the
plane of the paper as shown. The variation of the
figure. Then (2001)
magnetic field B along the line XX’ is given by (2000)
A
B

x x’ x x

d d (A) mA v A < mB vB (B) mA v A > mB vB


d d
(a) (b) (C) mA < mB and v A < =
vB (D) mA m
=B and v A vB

d
2 1 . 5 0 | Moving Charges and Magnetism

Q.7 A long straight wire along the z-axis carries a current (A) I > III > II > IV (B) I > II > III > IV
Iin the negative z-direction. The magnetic vector field
(C) I > IV > II > III (D) III > IV > I > II
B at a point having coordinate (x,y) on the z=0 plane is
 (2002)

(A)
(
ˆ ˆ
µ0 I yi − xj
(B)
) ˆ ˆ
µ0 I xi − yj ( ) Q.11 An electron moving with a speed u along the
position x-axis at y=0 enters a region of uniform
2π(x2 + y 2 ) 2π(x2 + y 2 ) 
magnetic field B = −B0kˆ which exists to the right of

(C)
(
µ0 I xjˆ − yiˆ ) (D)
(
µ0 I xiˆ − yjˆ ) y-axis. The electron exits from the region after sometime
2
2π(x + y ) 2
2π(x + y ) 2 2 with the speed v at coordinate y, then (2004)

y
Q.8 A particle of mass m and charge q moves with a
constant velocity v along the positive x-direction. It
enters a region containing a uniform field B directed e- u
along the negative z-direction, extending from x=a x
to x=b. the minimum value of v required so that the
particle can just enter the region x>b is  (2002)

q (b − a ) B qaB q (b + a ) B (A) v>u, y<0


(A) qbB (B) (C) (D) (B) v=u, y>0
m m m 2m
(C) v>u, y>0 (D) v=u, v<0
Q.9 For a positively charged particle moving in a x-y 
plane initially along x-axis, there is a sudden change Q.12 A magnetic field B = −B0 ˆj exists in the region

in its path due to presence of electric and/or magnetic a<x<2a and B = −B0 ˆj , in the region 2a<x<3a, where B0
fields beyond P. The curved path is shown in the x-y
plane and is found to be non-circular. is a positive constant. A positive point charge moving

Which one of the following combinations is possible? with a velocity v = − v 0 ˆi , where v0 is a positive constant,
 y (2003) enters the magnetic field at x=a.
B0

P x
O
0 x
a 2a 3a
   
(A) E= 0;B= bjˆ + ckˆ (B) = ˆ =
E ai; B ckˆ + aiˆ
    -B0
(C) E= 0;B= cjˆ + bkˆ (D) = ˆ =
E ai; B ckˆ + bjˆ
The trajectory of the charge in this region can be like
 (2007)
Q.10 A current carrying loop is placed in a uniform
magnetic field in four different orientations, I, II, III and z z
IV, arrange them in the decreasing order of potential
energy  (2003) a 2a 3a
(A) x (B) x
a 2a 3a


n B B
z z

n
(I) (II)
(C) (D) a 2a 3a
 x x
n a 2a 3a

B B

n
(III) (IV)
P hysi cs | 21.51

Q.13 Which of the field patterns given in the figure Q.17 The coercivity of a small magnet where the
is valid for electric field as well as for magnetic field? ferromagnet gets demagnetized is 3 × 103 Am−1 . The
 (2011) current required to be passed in a solenoid of length 10
cm and number of turns 100, so that the magnet gets
demagnetized when inside the solenoid, is: (2014)
(A) (B) (A) 3A (B) 6A (C) 30 mA (D) 60 mA

Q.18 A rectangular loop of sides 10 cm and 5 cm carrying


a current I of 12 A is placed in different orientations as
shown in the figures below:
z z
(C) (D) I
B B
(A) (C)
I I I
y y
I
Q.14 A long insulated copper wire is closely wound as a x x

spiral of N turns. The spiral has inner radius a and outer z


radius b. The spiral lies in the X-Y plane and a steady z
current I flows through the wire. The Z-component of B
I
the magnetic field at the center of the spiral is  (2011) (B)
B
I (D)
I y
I y I
x
µ0NI b µ0NI b +a I I
(A) ln   (B) ln   x I
2(b − a)  a  2(b − a)  b − a 
z z
µ0NI
b µ NI  b + a 
(C) ln   (D) 0 lnI 
2b  a  B
2b  b − a  B
(A) (C)
I I I
Q.15 Proton, Deuteron and alpha particle of the same y y
kinetic energy are moving in circular I trajectories in a
x
constant magnetic field. The
x
radii of proton, deuteron
and alpha particle are respectively rp, rd and ra. Which z
one of the following relations is correct?
z  (2012)
B
(A) rα= rp= rd (B) rαB= rp < rd I

(B) I (D)
I y
(C) rα > rd > rp (D) Irα= rd > rp y I
I
x I
x I
Q.16 Two short bar magnets of length 1 cm each have If there is a uniform magnetic field of 0.3 T in the positive
magnetic moments 1.20 Am2 and 1.00 Am2 respectively. z direction , in which orientations the loop would be
They are placed on a horizontal table parallel to each in (i) stable equilibrium and (ii) unstable equilibrium?
other with their N poles pointing towards the South.  (2015)
They have a common magnetic equator and are
separated by a distance of 20.0 cm. The value of the (A) (a) and (c), respectively
resultant horizontal magnetic induction at the mid (B) (b) and (d), respectively
- point O of the line joining their centres is close to
(Horizontal component of earth’s magnetic induction is (C) (b) and (c), respectively
3.6 × 10−5 Wb / m2 ) (2013) (D) (a) and (b), respectively
−4 2 −4 2
(A) 2.56 × 10 Wb / m (B) 3.50 × 10 Wb / m
(C) 5.80 × 10−4 Wb / m2 (D) 3.6 × 10−5 Wb / m2
2 1 . 5 2 | Moving Charges and Magnetism

JEE Advanced/Boards

Exercise 1 when a current of 1amp enters in the loop and taken


out of it by two long wires as shown in the figure.

Q.1 A system of long four parallel conductors whose


sections with the plane of the drawing lie at the vertices
of a square there flow four equal currents. The directions O
of these currents are as follows:  45
o

1amp 90
o
I1 I2
1amp


Q.5 Find the magnetic induction at the origin in the


figure shown.
y
Those marked ⊗ point away from the reader, while
those marked with a dot point towards the reader.
How is the vector of magnetic induction directed at the i A i
center of the square? z i
i
x
Q.2 A long straight wire carriers a current of 10A directed
along the negative y-axis as shown in figure. A uniform
Q.6 Find themagnetic induction at point O, if the current
magnetic field B0 of magnitude 10-6T is directed parallel
carrying wire is in the shape shown in the figure.
to the x-axis. What is the resultant magnetic field at the
following points?
r I
z O
y r

Q.7 Find the magnitude of the magnetic induction


B of a magnetic field generated by a system of thin
x conductors along which a current I is flowing at a point
A(O,R,O), that is the center of a circular conductor of
i radius R. The ring is in the yz plane.

Y
(a) x=0, z=2m ;
(b) x=2m, z=0 R

(c) x=0, z=-0.5m R RR


X
R
Q.3 Find the magnetic field at the center P of square of I
side a shown in figure. Z

Q.8 A cylindrical conductor of radius R carriers a current


along its length. The current density J, however, is not
P uniform over the cross section of the conductor but is
i i a function of the radius according to J=br, where b is a
constant. Find an expression for the magnetic field B.
Q.4 What is the magnitude of magnetic field at the
(a) at r1 < R (b) at distance r2 < R , measured from the
center ‘O’ of loop of radius 2 m made of uniform wire axis
P hysi cs | 21.53

i that is given by β = αy where α is constant. Find the


total magnetic force on the loop if it carries current i.
R
Y
a

Q.9 Electric charge q is uniformly distributed over a rod B


a
of length L. The rod is placed parallel to a long wire
carrying a current I. The separation between the rod X
and the wire is a. Find the force needed to move the rod
Q.16 A particle of charge +q and mass m moving
along its lengths with a uniform velocity V.
under the influence of a uniform electric field E î and a
magnetic field B k̂ enters in I quadrant of a coordinate
Q.10 An electron moving with a velocity 5x106 ms−1ˆj system at a point (0, a) with initial velocity v î and leaves
in the uniform electric field of 5x107 Vm−1ˆj . Find the the quadrant at a point (2a, 0) with velocity −2v ˆj . Find
magnitude and direction of a minimum uniform Magnitude of electric field
magnetic field in tesla that will cause the electron to (a) Rate of work done by the electric field at point
move undeviated along it original path.
(b) (0, a) Rate of work done by both the fields at.

Q.11 A charged particle (charge q, mass m) has velocity (c) (2a, 0).
V0 at origin in +x direction. In space there is a uniform
magnetic field B in –z direction. Find the y coordinate of Q.17 A square current carrying loop made of thin wire
the particle when it crosses y axis. and having a mass m=10g can rotate without friction
with respect to the vertical axis OOI , passing through
Q.12 A proton beam passes without deviation through the center of the loop at right angles to two opposite
a region of space where there are uniform transverse sides of the loop. The loop is placed in a uniform
mutually perpendicular electric and magnetic field with magnetic field with an induction B=10-1T directed at
E and B. Then the beam strikes a grounded target. Find right angles to the plane of the drawing. A current
the force imparted by a beam on the target if the beam I=2A is flowing in the loop. Find the period of small
current is equal to I. oscillations that the loop performs about its position of
stable equilibrium.

Q.13 A conducting circular loop of radius r carriers a O B


constant current i. It is placed in a uniform magnetic
field B0 such that B0 is perpendicular to the plane of the
i
loop. Find the magnetic force acting on the loop.

Q.14 An arc of a circular loop of radius R is kept in the O1


horizontal plane and a constant magnetic field B is
applied in the vertical direction as shown in the figure. Q.18 An infinitely long straight wire carries a
If the carries current I then find the force on the arc. conventional current I as shown in the figure. The
rectangular loop carries a conventional current I’ in
x x x x x
x the clockwise direction. Find the net force on the
Bx
x x x x x rectangular loop.
I
x x x x x x

x x x o
x x x
90
x x x x x x
i’ c
x x x x x x

a
Q.15 A rectangular loop of wire is oriented with the
left corner at the origin, one edge along X-axis and b
the other edge along. Y-axis as shown in the figure. A
magnetic field is into the page and has a magnitude
2 1 . 5 4 | Moving Charges and Magnetism

Q.19 3 Infinitely long thin wires each carrying current i (b) If an external uniformmagnetic induction field
in the same direction, are in the x-y plane of a gravity B = Biˆ is applied, find the torque acting on the loop
free space. The central wire is along the y-axis while due to the field.
the other two are along x = ±d. (i) Find the locus of the
points for which the magnetic field B is zero. Q.23 (a) A rigid circular loop of radius r & mass m lies in
(ii) If the central wire is displaced along the Z-direction the xy plane on a flat table and has a current I flowing
by a small amount & released, show that it will execute in it. At this particular place, the earth’s magnetic field
simple harmonic motion. If the linear density of the B Bx ˆi + B y ˆj . How large must I be before one edge
is =
wires is λ , find the frequency of oscillation. of the loop will lift from table?
B Bx ˆi + Bzk.
(b) Repeat if,= ˆ
Q.20 Q charge isuniformly distributed over the same
surface of a right circular cone of semi-vertical angle Q.24 A conductor carrying a current is placed parallel
θ and height h. The cone is uniformly rotated about a current per unit width j0 and width d, as shown in the
its axis at angular velocity ω . Calculated associated Figure.
magnetic dipole moment.
z


 j0 h
h d  y

Q.21 Four long wires each carrying current I as shown


Find the force per unit length on the conductor.
in the figure are placed at the point A, B, C and D. Find
the magnitude and direction of
Q.25 The figure shows a conductor of weight 1.0N and
Y length L= 0.5m placed on a rough inclined plane making
an angle 300 with the horizontal so that conductor is
D(-a,a) + A(a,a)
perpendicular to a uniform horizontal magnetic field
X of induction B=0.10 T. The coefficient of static friction
C(-a,a) + B(a,-a) between the conductor and the plane is 0.1. A current
of I=10A flows through the conductor inside the plane
of this paper as shown. What is the force that should be
(i) Magnetic field at the center of the square. applied parallel to the inclined plane for sustaining the
conductor at rest?
(ii) Force per metre acting on wire at point D.

Q.26 An electron gun G emits electron of energy 2kev


Q.22 A wire loop carrying current I is placed in the X-Y
traveling in the (+) ve x-direction. The electron are
plane as shown in the figure.
required to hit the spot S where GS=0.1m & line GS
M makes an angle of 600 with the x-axis, as shown in the
Y figure. A uniform magnetic field B parallel to GS exists
a
V in the region outside to the electron gun. Find the
minimum value of B needed to make the electron hit S.
120o

I I P
a S
X B
N B
o
60
(a) If a particle with charge +Q and mass m is placed at
Gun X
the center P and given a velocity along NP (see figure).
Find its instantaneous acceleration.
P hysi cs | 21.55

Q.27 Two coils each of 100 turns are held such that one Exercise 2
lies in the vertical plane with their centers coinciding.
The radius of the vertical coil is 20cm and that of the Single Correct Choice Type
horizontal coil is 30cm. How would you neutralize the
magnetic field of the earth at their common center?
What is the current to be passed through each coil? Q.1 Two very long straight parallel wires, parallel to -y
Horizontal component of earth’s magnetic induction = direction, respectively. The wire are passes through the
3.49 x 10-5T and angle of dip=300. x-axis at the point (d, 0, 0) and (-d, 0, 0)respectively. The
graph of magnetic field z-component as one moves
along the x-axis from x=-d to x=+d, is best given by
Q.28 An infinite wire, placed along z-axis, has current
i1 in positive z-direction. A conducting rod placed in o x
xy plane parallel to y-axis has current i2 in positive (A) (B)
y-direction. The ends of the rod subtend +300 and -600 o
at the origin with positive x-direction. The rod is at a
distance a from the origin. Find net force on the rod.
(C) x (D) o x
o
Q.29 A square loop of wire of edge a carries a current i.
(a) Show that B for a point on the axis of the loop and a Q.2 A long thin walled pipe of radius R carries a current
distance x from its center is given by, I along its length. The current density is uniform over
4µ0ia2 the circumference of the pipe. The magnetic field at the
B= center of the pipe due to quarter portion of the pipe
( )( 4x )

1/2
π 4x2 + a2 2
+ 2a2 shown, is

(b) Can the result of the above problem be reduced to


give field at x=0?

Q.30 A straight segment OC (of length L meter) of a µ0 I 2 µ0 I


circuit carrying a current I amp is placed along the (A) (B)
4π R 2
π2R
x-axis. Two infinitely line straight wires A and B, each
extending from z = −∞ to + ∞ , are fixed by y=−a 2µ0 I 2
(C) (D) None
meter and y=+a meter respectively, as shown in the π2R
Figure.

Q.3 An electron (mass=9.1 x 10-31; charge=-1.6 x 10-19C)


y
experiences no deflection if subjected to an electric field
B of 3.2 x 105 V/m and a magnetic field of 2.0 x 10-3 Wb/
m2. Both the fields are normal to the path of electron
and to each other. If the electric field is removed, then
O x the electron will revolve in an orbit of radius:
C
(A) 45m (B) 4.5m (C) 0.45m (D) 0.045m
z A
Q.4 A particle of specific charge (charge/mass) α starts
moving from the origin under the action of an electric
If the wires A and B each carry a current I amp into
plane of the paper. Obtain the expression for the force field E = E0 ˆi and magnetic field B = B0kˆ . Its velocity at
acting on the segment OC. What will be the force OC if
current in the wire B is reversed?
( x , y 0 ) is ( 4iˆ − 3ˆj) . The value of x
0 0, 0
is:

13 αE0 16αB0
(A) (B)
2 B0 E0

25 5α
(C) (D)
2αE0 2B0
2 1 . 5 6 | Moving Charges and Magnetism

Q.5 A particle of specific charge (q/m) is projected from Q.9 A conducting wire bent in the form of a parabola
the origin of coordinates with initial velocity [ui-vj]. y 2 = 2x carriers a current i=2A as shown in figure. This
Uniform electric magnetic field exist in the region along wire is placed in a uniform magnetic field B = −4kˆ Tesla.
the +y direction, of magnitude E and B. The particle will The magnetic force on the wire is (in newton).
definitely return to the origin once if y(m) A
(A)  vB / 2πE  is an integer

( )
1/2 2 x(m)
(B) u2 + v 2 B / πE  is an integer
B
(C)  vB / πE  in an integer
(A) −16iˆ (B) 32iˆ
(D) uB / πE  is an integer.
(C) −32iˆ (D) 16iˆ

Q.6 Two particles of charges +Q and –Q are projected Q.10 A semicircular current carrying wire having radius
from the same point with a velocity v in a region of R is placed in x-y plane with its center at origin ‘O’.
uniform magnetic field B such that the velocity vector There is non-uniform magnetic field
makes an angle θ with the magnetic field. Their masses  B x
are M and 2M, respectively. Then, they will meet again B = o kˆ (here Bo is +ve constant) is existing in the
for the first time at a point whose distance from the 2R
region. The magnetic force acting on semicircular wire
point of projection is
will be along
(A) 2πMv cos θ / QB (B) 8πMv cos θ / QB Y
(C) πMv cos θ QB (D) 4 πMv cos θ / QB
i
Q.7 A particle with charge +Q and mass m enters a
magnetic field of magnitude B, existing only to the (-R,0,0) (+R,0,0) X
right of the boundary YZ. The direction of the motion
of the particle is perpendicular to the direction of B. Let Z
(A) –x-axis (B) +y-axis
2πM
T= . The time spent by the particle in the field
QB (C) –y-axis (D) +x-axis
will be
Q.11 A square loop ABCD, carrying a current I, is placed
(A) T θ (B) 2T θ
near and coplanar with a long straight conductor XY
 π + 2θ   π − 2θ  carrying a current I, the net force on the loop will be
(C) T   (D) T  
 2π   2π 
B C
+Q
Y
m x B i L
x A D
 x L/2 L
x
2µ0 Ii µ0 Ii 2µ0 Iil µ0 Iil
x (A) (B) (C) (D)
Z 3π 2π 3π 2π
Q.8 In the previous question, if the particle has-Q
charge, the time spend by the particle in the field will Q.12 A conducting ring of mass 2kg and radius 0.5m is
be placed on a smooth horizontal plane. The ring carries
a current i=4A. A horizontal magnetic field B=10T is
(A) T θ (B) 2T θ
switched on at time t=0 as shown in figure. The initial
 π + 2θ   π − 2θ  angular acceleration of the ring will be
(C) T   (D) T  
 2π   2π 
P hysi cs | 21.57

(A) Its x-coordinate can never be positive


 (B) Its x- and z-coordinates cannot both be zero at the
B
same time.
(C) Its z-coordinate can never be negative.
(D) Its y-coordinate will be proportional to the square
(A) 40π rad / s2 (B) 20π rad / s2 of its time of flight.
(C) 5π rad / s2 (D) 15π rad / s2
Multiple Correct Choice Type
Q.13 In the following hexagons, made up of two
different material P and Q, current enters and leaves Q.16 Which of the following statements is correct:
from points X and Y respectively. In which case the (A) A charged particle enters a region of uniform
magnetic field at its center is not zero. magnetic field at an angle 850 to magnetic lines of
Q
Y
Q
Y
force. The path of the particle is a circle.
Q P
P P (B) An electron and proton are moving with the same
(A) (B) kinetic energy along the same direction. When they
P Q Q Q pass through uniform magnetic field perpendicular to
X
P
X
P their direction of motion, they describe circular path.
(C) There is no change in the energy of a charged
Q P particle moving in a magnetic field although magnetic
Y Y
P Q force acts on it.
Q Q
(C) (D) (D) Two electrons enter with the same speed but in
P Q P Q opposite direction in a uniform transverse magnetic
X
P
X
P field. Then the two describe circle of the same radius
and these move in the same direction.
Q.14 Current flows through uniform, square frames as
shown. Q.17 Consider the magnetic field produced by a finitely
long current carrying wire.
In which case is the magnetic field at the center of the
frame not zero? (A) The lines of field will be concentric circles with
centers on the wire.
(B) There can be two points in the same plane where
(A) (B) magnetic fields are same.
(C) There can be large number of points where the
magnetic field is same.

(C) (D) (D) The magnetic field at a point is inversely proportional


to the distance of the point from the wire.

Q.15 In a region of space, a uniform magnetic field Q.18 A long straight wire carriers a current along the
B exists in the y-direction. A proton is fired from the x-axis. Consider the points A(0,1,0), B(0,1,1), C(1,0,1)
origin, with initial velocity v making a small angle α and D(1,1,1). Which of the following pairs of points will
with the y-direction in the yz plane. In the subsequent have magnetic field of the same magnitude?
motion of the proton, z (A) A and B (B) A and C (C) B and C (D) B and D

B
Q.19 Consider three quantities x=E/B, = y 1 / µ0 ε0
1
v and z = . Here, l is the length of a wire, C is a
CR

y capacitance and R is a resistance. All other symbols
O B
have standard meanings.
x
2 1 . 5 8 | Moving Charges and Magnetism

(A) x,y have the same dimensions (B) While deflecting in magnetic field its energy
gradually increases.
(B) y, z have the same dimension
(C) Only the component of magnetic field perpendicular
(C) z, x have the same dimensions
to the direction of motion
(D) None of the three pairs have the same dimensions.
of the charged particle is effective in deflecting it.
(D) Direction of deflecting force on the moving charged
Q.20 Two long thin, parallel conductors carrying equal
particle is perpendicular to its velocity.
currents in the same direction are fixed parallel to the
x-axis, one passing through y=a and the other through
Assertion Reasoning Type
y=-a. The resultant magnetic field due to the two
conductors at any point is B. Which of the following (A) Statement-I is true, statement-II is true and
are correct? Statement-II is correct explanation for Statement-I.
z (B) Statement-I is true, statement-II is true and statement-
II is NOT the correct explanation for statement-I.
(C) Statement-I is true, statement-II is false.
-a (D) Statement-I is false, statement-II is true.
O a y
i
i Q.24 Statement-I: A charged particle can never move
x
along a magnetic field line in absence of any other
force.
(A) B=0 for all points on the x-axis Statement-II: Force due to magnetic field is given by
  
(B) At all points on the y-axis, excluding the origin, B ( )
F = q v xB .
has only a z-component.
(C) At all points on the z-axis, excluding the origin, B Q.25 Statement-I : It is not possible for a charged
has only an x-component. particle to move in a circular path around a long
straight uncharged conductor carrying current under
Q.21 An electron is moving along the positive X-axis. the influence of its magnetic field alone.
You want to apply a magnetic field for a short time so Statement-II: The magnetic force (if nonzero) on a
that the electron may reverse its direction and move moving charged particle is normal to its velocity.
parallel to the negative X-axis. This can be done by
applying the magnetic field along.
Q.26 Statement-I: For a charged particle to pass through
(A) Y-axis (B) Z-axis a uniform electro-magnetic field without change in
velocity, its velocity vector must be perpendicular to
(C) Y-axis only (D) Z-axis only
the magnetic field.

Q.22 Two identical charged particles enter a uniform Statement-II: Net Lorentz force on the particle is given

magnetic field with same speed but at angles 300 and by=F q E + v xB 
600 with field. Let a, b and c be the ratio of their time
periods, radii and pitches of the helical paths then Q.27 Statement-I: Two long parallel conductors
carrying current in the same direction experience a
(A) abc=1 (B) abc > 1
force of attraction.
(C) abc < 1 (D) a=bc
Statement-II: The magnetic fields produced in the
space between the conductors are in the same direction.
Q.23 Consider the following statements regarding
a charged particle in a magnetic field. Which of the
Q.28 Statement-I: Ampere law can be used to find
statement are true :
magnetic field due to finite length of a straight current
(A) Starting with zero velocity, it accelerates in a carrying wire.
direction perpendicular to the magnetic field.
P hysi cs | 21.59

Statement-II: The magnetic field due to finite length Q.30 The magnetic field at C due to curved part is
of a straight current carrying wire is symmetric about µ0 I
the wire. (A) , directed into the plane of the paper

µ0 I
Q.29 Statement-I: A pendulum made of a non- (B) , directed towards you

conducting rigid massless rod of length  is attached to
a small sphere of a mass m and charge q. The pendulum µ0 I
(C) , directed towards you
is undergoing oscillations of small amplitude having 3α
time period T. Now a uniform horizontal magnetic field µ0 I
(D) , directed up the plane of the paper
out of plane of page is switched on. As a result of this 3α
change, the time period of oscillations will change.

 Q.31 A wire loop carrying a current I is shown in figure.


B The magnetic field induction at C due to straight part

is

m,q
a

120o
C
Statement-II: In the situation of statement-I, after the I I
magnetic field is switched on the tension in string will a
change (except when the bob is at extreme position).

Comprehension Type 3µ0 I


(A) , directed up the plane of the paper
2πα
Paragraph 1: Magnetic field intensity (B) due to current
µ0 I
carrying conductor can be calculated by use of Biot- (B) , directed into the plane of the paper
Savart law. Which is 6α
µ0 Idlxr µ0 I
B= , (C) , directed towards you
4π r3 6α
µ0 I  3 1 
(D)  −  towards you
R 2α  π 3 
I 
I  O R 
Q.32 The net magnetic field at C due to the current
carrying loop is directed into the plane of the paper
µ0 I
(A) Zero (B)
α
where dB is magnetic field due current element Idl at
µ0 I µ I 3µ0 I
a position r from current element. For straight wire (C) (D) B =
− 0 + ,
carrying current magnetic field at a distance R from 9α 6a 2πa
wire is
µ0 I Paragraph 2: A current carrying coil behave like short
=B
4π R
( sin α + sinβ ) magnet whose magnetic dipole moment M=nIA. Where
direction of M is taking along the direction of magnetic
And magnetic field due to a circular arc at its center is fields on its axis and n is no of turns A is area of coil and
µ0 I I is current flowing through coil. When such a coil is put
=B .θ in magnetic field (B) magnetic torque ( τ ) acts on it as
4 πR
τ = −MxB and potential energy of the current loop in
where θ angle of circular arc at center, R is radius of
the magnetic field is u=-M.B.
circular arc.
2 1 . 6 0 | Moving Charges and Magnetism

Q.33 A current of 3A is flowing in a plane circular coil of Q.35 In above question, to hold the current-carrying
radius 1cm and having 20 turns. The coil is placed in a coil with the normal to its plane making an angle of 900
uniform magnetic field of 0.5 Wbm-2. Then, the dipole with the direction of magnetic induction, the necessary
moment of the coil is torque is
(A) 3000Am2 (B) 0.3Am2 (A) 1500 Nm (B) 9.4 x 10-3 Nm
(C) 75 Am2 (D) 1.88x10−2 Am2 (C) 15 Nm (D) 150 Nm

Q.34 A current of 3A is flowing in a plane circular coil


of radius 1cm and having 20 turns. The coil is placed
in a uniform magnetic field of 0.5 Wbm-2. Then, the
P.E. of the magnetic dipole in the position of stable
equilibrium is
(A) -1500 J (B) -9.4 mJ
(C) +0.15 J (D) +1500 J

Match the Column

Q.36 Two wires each carrying a steady current I are shown in four configuration in column I. Some of the resulting
effects are described in column II. Match the statement in column I with the statements in column II and indicate
your answer by darkening appropriate bubbles in the 4 x 4 matrix given in the ORS.

Column I Image Column II


(A) Point P is situated midway (p) The magnetic fields (B) at P due to
between the wires the currents in the wires are in the same
P
direction.

(B) Point P is situated at the mid- (q) The magnetic fields (B) at P due to
point of the line joining the centers the current in the wires are in opposite
of the circular wires, which have directions.
same radii. P

(C) Point P is situated at the (r) There is no magnetic field at P.


mid-point of the line joining the
P
centers of the circular wires, which
have same radii.
P hysi cs | 21.61

Q.37 Six point charges, each of the same magnitude q, are arranged in different manners as shown in column II. In
each case, a point M and a line PQ passing through M are shown. Let E be the electric field and V be the electric
potential at M (potential at infinity is zero) due to the given charge distribution when it is at rest. Now, the whole
system is set into rotation with a constant angular velocity about the line PQ. Let B be the magnetic field at M and
µ be the magnetic moment of the system in this condition. Assume each rotating charge to be equivalent to a
steady current.

Column-I Image Column-II


(A) E=0 Charges are at the corners of a regular
+ - Q hexagon. M is the center of the hexagon. PQ
is perpendicular to the plane of the hexagon.
- +
M

P + -

(B) V ≠ 0 Charges are on a line perpendicular to PQ at


P
equal intervals. M is the midpoint between
- + - + - + the two innermost charges.
M

(C) B=0 Charges are placed at the corners of a


rectangle of sides a and 2a and at the mid
+ - +
Q points of the longer sides. M is at the center
of the rectangle. PQ is parallel to the longer
- M - sides.

P
+

- - Charges are placed at the corners of a


(D) µ ≠ 0 + rectangle of sides a and 2a and at the mid
M points of the longer sides. M is at the center
P Q of the rectangle. PQ is parallel to the longer
sides.
- + -
2 1 . 6 2 | Moving Charges and Magnetism

Previous Years’ Questions Q.3 A superconductor has TC(0)=100K . When a


magnetic field of 7.5 Tesla is applied, its TC decreases to
Q.1 Statement I: The sensitivity of a moving coli 75K. For this material one can definitely say that when
galvanometer is increased by placing a suitable (Note: T=Tesla)  (1987)
magnetic material as a core inside the coil.  (2008) (A) B=5T, TC(B)=80K
Statement II: Soft iron has a high magnetic permeability (B) B=5T, 75K < TC(B) < 100K
and cannot be easily magnetized or demagnetized.
(C) B=10T, 75K < TC(B) < 100K
(A) Statement-I is true, statement-II is true and
(D) B=10T, TC(B)=70K
Statement-II is correct explanation for Statement-I.
(B) Statement-I is true, statement-II is true and statement-
Q.4 A proton moving with a constant velocity passes
II is NOT the correct explanation for statement-I.
through a region of space without any change in its
(C) Statement-I is true, statement-II is false. velocity. If E and B represent the electric and magnetic
fields respectively. Then, this region of space may have
(D) Statement-I is false, statement-II is true.
 (1985)
Passage: (Q.2-Q.3) (A) E=0, B=0 (B) E=0, B≠0
Electrical resistance of certain material, known as (C) E≠0, B=0 (D) E≠0, B≠0
superconductors, changes abruptly from a non-zero
value to zero as their temperature is lowered below
Q.5 A particle of charge
a critical temperature TC ( 0 ) . An interesting property y 
+q and mass m moving v E
of superconductors is that their critical temperature P
under the influence of a 
becomes smaller than TC ( 0 ) if they are placed in a
uniform electric field Eiˆ
B
magnetic field i.e., the critical temperature TC (B ) is a
and uniform magnetic a
function of the magnetic strength B. The dependence
field Bkˆ follows a
of TC (B ) on B is shown in the Figure. Q
trajectory from P to Q as 2a 2v
x
shown in Figure. The
TC(B)
velocities at P and Q are
viˆ and − 2ˆj . Which of the following statement (s) is/are
TC(0) correct? (1991)

3  mv 2 
(A) E =  
4  qa 
O B
(B) Rate of work done by the electric field at P is
Q.2 In the graphs below, the resistance R of a
superconductor is shown as a function of its temperature 3  mv 2 
T for two different magnetic fields B1 (solid line) and  
4  a 
B2 (dashed line). If B2 is larger than B1 , which of the
following graphs shows the correct variation of R with (C) Rate of work done by the electric field at P is zero
T in these fields? (2010)
(D) Rate of work done by both the fields at Q is zero
R R B2
B1 Q.6 H+ ,He+ and O2+ all having the same kinetic energy
(A) (B) pass through a region in which there is a uniform
magnetic field perpendicular to their velocity. The
T T
masses of H+ ,He+ and O2+ are 1 amu, 4 amu and 16
O B2 B1 O
amu respectively. Then (1994)

R R (A) H+ will, be deflected most


B1
B2 (B) O2+ will be deflected most
(C) (D)
B1 B2
(C) He+ and O2+ will be deflected equally
O T O T
(D) All will be deflected equally
P hysi cs | 21.63

Q.7 Which of the following statement is (are) correct in (A) They will never come out of the magnetic field
the given Figure? (2006) region

infinitely long wire kept perpendicular (B) They will come out travelling along parallel axis
C
to the paper carrying current inwards B (C) They will come out at the same time
l1
l2 (D) They will come out at different times.
O O’
Q.10 Consider the motion of a positive point charge in
a region where there are
 simultaneous  uniform electric
A and magnetic fields E = E0 ˆj and B = B0 ˆj . At time t =

D 0, this charge has velocity v in the x-y plane, making
(A) Net force on the loop is zero. an angle θ with the x-axis. Which of the following
option(s) is(are) correct for time t > 0 ? (2012)
(B) Net torque on the loop is zero.
(A) If θ =0o , the charge moves in a circular path in the
(C) Loop will rotate clockwise about axis OO' when x-z plane.
seen from O
(B) If θ =0o , the charge undergoes helical motion with
(D) Loop will rotate anticlockwise about OO’ when seen constant pitch along the y-axis.
from O
(C) If θ =10o , the charge undergoes helical motion
with its pitch increasing with time, along the y-axis
Q.8 A particle of mass m and charge q. moving with
velocity v enters Region II normal to the boundary as (D) θ =90o , the charge undergoes linear but accelerated
shown in the Figure. Region II has a uniform magnetic motion along the y-axis.
field B perpendicular to the plane of the paper. The
length of the Region II is l. Choose the correct choice Q.11 A cylindrical cavity of diameter a exists inside
(s).  (2008) a cylinder of diameter 2a as shown in the figure.
Both the cylinder and the cavity are infinitely long. A
Region I Region II Region III
uniform current density J flows along the length. If the
x x x x magnitude of the magnetic field at the point P is given
x x x x N
by µ aJ , then the value of N is  (2012)
v
x x x x 12 0
x x x x
x x x x

a
l P O
qlB
(A) The particle enters Region III only if its velocity> .
m
(B) The particle enters Region III only if its velocity 2a
qlB Q.12 A loop carrying current I lies in the x-y plane as
v< .
m shown in the figure. The unit vector k̂ is coming out of
(C)Path length of the particle in Region II is maximum the plane of the paper. The magnetic moment of the
qlB current loop is - (2012)
when velocity v= . y
m
(D) Time spent in Region II is same for any velocity v as
long as the particle returns to Region I.
I
Q.9 An electron and a proton are moving on straight a x
parallel paths with same velocity. They enter a semi-
infinite region of uniform magnetic field perpendicular
a
to the velocity. Which of the following statement(s) is/
are true? (2011)
2 1 . 6 4 | Moving Charges and Magnetism

π  50πM
(C) The magnitude of the magnetic field units.
(A) a2 I kˆ (B)  + 1  a2 I kˆ 3Q
2 
100πM
(D) The magnitude of the magnetic field is
π  units 3Q
(C) −  + 1  a2 I kˆ (D) (2π + 1)a2 I kˆ
2 
Q.15 Two bodies, each of mass M, are kept fixed with a
Q.13 An infinitely long hollow conducting cylinder with separation 2L. A particle of mass m is projected from the
inner radius R/2 and outer radius R carries a uniform midpoint of the line joining their centres, perpendicular
current density along
 its length. The magnitude of the to the line. The gravitational constant is G. The correct
magnetic field, | B | as a function of the radial distance statement(s) is (are) (2013)
r from the axis is best represented by  (2012)
(A) The minimum initial velocity of the mass m to escape
GM
the gravitational field of the two bodies is 4
  L
(A) B (B) B
(B) The minimum initial velocity of the mass m to escape
GM
R/2 R the gravitational
R/2 R field of the two bodies is 2
L

  (C) The minimum initial velocity of the mass m to escape


(B) BB
(A) (B) B

B
(D) 2GM
(C) B the gravitational field of the two bodies is
L
R/2
R/2 RR
r (D) TheR
R/2 Renergy of the mass m remains constant.
R/2 R
Q.16 Two parallel wires in the plane of the paper are
  distance X0 apart. A point charge is moving with speed
(D) B  (D) B
(C) B u between the wires in the same plane at a distance X1
(B) B from one of the wires. When the wires carry current of
magnitude I in the same direction, the radius of curvature
r r R
R/2 R
R of the path of the point charge is R1. In contrast, if the
R/2 R currents I in the two wires have directions opposite to
each other, the radius of curvature of the path is R 2 .
X0 R1
 If = 3 , the value of is (2014)
(D) B X1 R2

Q.17 When d ≈ a but wires are not touching the loop,


r it is found that the net magnetic field on the axis of the
R
loop is zero at a height h above the loop. In that case
 (2014)
Q.14 A particle of mass M and positive charge Q, moving
with a constant velocity u1 = 4imsˆ −1 , enters a region of (A) Current in wire 1 and wire 2 is the direction PQ and
uniform static magnetic field normal to the x-y plane. RS, respectively and h ≈ a
The region of the magnetic field extends from x = 0 (B) Current in wire 1 and wire 2 is the direction PQ and
to x = L for all values of y. After passing through this SR, respectively and h ≈ a
region, the particle emerges on the other side after 10
 (C) Current in wire 1 and wire 2 is the direction PQ and
milliseconds with a velocity= u2 2( 3iˆ + ˆj)m / s−1 . The
SR, respectively and h ≈ 1.2a
correct statement(s) is (are)  (2013)
(D) Current in wire 1 and wire 2 is the direction PQ and
(A) The direction of the magnetic field is -z direction.
RS, respectively and h ≈ 1.2a
(B) The direction of the magnetic field is +z direction.
P hysi cs | 21.65

Q.18 Consider d >> a, and the loop is rotated about its Q.20 Consider two different metallic strips (1 and 2) of
diameter parallel to the wires by 30° from the position the same material. Their lengths are the same, widths
shown in the figure. If the currents in the wires are in are w1 and w2 and thicknesses are d1 and d2, respectively.
the opposite directions, the torque on the loop at its Two points K and M are symmetrically located on the
new position will be (assume that the net field due to opposite faces parallel to the x-y plane (see figure). V1
the wires is constant over the loop) (2014) and V2 are the potential differences between K and M
in strips 1 and 2, respectively. Then, for a given current I
µ0 I2a2 µ0 I2a2 flowing through them in a given magnetic field strength
(A) (B)
d 2d B, the correct statement(s) is(are)  (2015)
(A) If w1 = w2 and d1 = 2d, then V2 = 2V1
3µ0 I2a2 3µ0 I2a2
(C) (D) (B) If w1 = w2 and d1 = 2d2, then V2 = V1
d 2d
(C) If w1 = 2w2 and d1 = d2, then V2 = 2V1
Q.19 A conductor (shown in the figure) carrying constant (D) If w1 = 2w2 and d1 = d2, then V2 = V1
current
 I is kept in the x-y plane in a uniform magnetic
field B . If F is the magnitude of the total magnetic force Q.21 Consider two different metallic strips (1 and 2)
acting on the conductor, then the correct statement(s) of same dimensions (lengths  , with w and thickness
is (are). (2015) d) with carrier densities n1 and n2, respectively. Strip
y 1 is placed in magnetic field B1 and strip 2 is placed
in magnetic field B2, both along positive y-directions.
R R Then V1 and V2 are the potential differences developed
I /6 /4
x between K and M in strips 1 and 2, respectively.
L R R L Assuming that the current I is the same for both the
 strips, the correct option(s) is(are) (2015)
(A) If B is along ẑ, F ∝ (L + R)
 (A) If B1 = B2 and n1 = 2n2, then V2 = 2V1
(B) If B is along x̂, F = 0
 (B) If B1 = B2 and n1 = 2n2, then V2 = V1
(C) If B is along ŷ, F ∝ (L + R)
 (C) If B1 = 2B2 and n1 = n2, then V2 = 0.5V1
(D) If B is along ẑ, F = 0
(D) If B1 = 2B2 and n1 = n2, then V2 = V1

MASTERJEE Essential Questions

JEE Main/Boards JEE Advanced/Boards


Exercise 1 Exercise 1
Q. 7 Q.8 Q.12 Q.4 Q.5 Q.16 Q.19
Q.20 Q.25 Q.22 Q.25 Q.30
Q.26 Q.27
Exercise 2
Exercise 2 Q.1 Q.3 Q.11 Q.13
Q.2 Q.5 Q.20 Q.15 Q.19 Q.20 Q.22
Q.40 Q.42 Q.44 Q.45
Q.48 Q.50
2 1 . 6 6 | Moving Charges and Magnetism

Answer Key

JEE Main/Boards
Exercise 1
Q.1 π × 10−4 T ≈ 3.1 × 10−4 T Q.2 3.5 × 10−5 T
Q.3 4 × 10−6 T, vertically up Q.4 1.2 × 10−5 T, towards south
Q.5 0.6N m−1 Q.6 4.2cm
Q.7 (i) 3.1 Nm, (ii) No Q.8 5π × 10−4=
T 1.6 × 10−3 T towards west
Q.9 (a) A horizontal magnetic field to magnitude 0.26T normal to the conductor in such a direction that Fleming’s
left-hand rule gives a magnetic force upward. (b) 1.176N
Q.10 1.22N m−1
Q.11 (a) 2.1 N vertically downwards (b) 2.1N vertically downwards (c) 1.68N vertically downwards
Q.12 2:1
Q.13 (a) Zero (b) Zero (c) Force on each electron in evB=IB(nA)= 5 × 10−25 N .
Note: Answer (c) Denotes only the magnetic force.
µ0 I µ0 I
Q.14 B = Q.15 B =
2R 2πR
Q.16 B = µ0 I N where N is the number of turns per unit length and I is the current flowing through the solenoid.
   
Q.17 2:1 Q.19 F =qE + q(v × B)
Q.20 Circle Q.21 4 × 10−5 T
Q.22 B=14.1 Wb Q.23 1.57 × 10−2 T
Q.24 3 × 10−12 Q.25 6 × 10−7 Nm−1
Q.26 5.9 × 10−6 N m Q.27 22.5V
Q.28 (i) 8A (ii) 3 × 10−7 T (iii) 7.68 × 10 –6 Nm−1

Exercise 2
Q.1 D Q.2 C Q.3 A Q.4 A Q.5 A Q.6 A
Q.7 B Q.8 D Q.9 D Q.10 B Q.11 A Q.12 B
Q.13 C Q.14 C Q.15 A Q.16 A Q.17 A Q.18 C
Q.19 D Q.20 B Q.21 B

Previous Years’ Questions


Q.1 D Q.2 C Q.3 B Q.4 D Q.5 C Q.6 B
Q.7 A Q.8 B Q.9 B Q.10 C Q.11 D Q.12 A
Q.13 C Q.14 A Q.15 B Q.16 A Q.17 A Q.18 B
P hysi cs | 21.67

JEE Advanced/Boards
Exercise 1
Q.1 In the plane of the drawing from right to left
Q.2 (a) 0 (b) 1.41 × 10−6 T, 45° in xz plane, (c) 5 × 10−6 T , +x-direction
(1 − 2 2)µ0 I
Q.3 k̂ Q.4 zero
πa
µ0 I  3 ˆ 1 ˆ  µ0i  3 
Q.5  k + j  Q.6  π + 1
4R  4 π  4 πR  2 

µ0i µ0br12 µ0br3


Q.7 B
=
4 πR
(
2 2π2 − 2π + 1 ) Q.8 B1
= =
3
, B2
3r2

µοiqv
Q.9 Q.10 10kˆ
2πa
2mv 0 mEI
Q.11 Q.12
qB Be

Q.13 Zero Q.14 2 IRB ˆj

3mv 2 3mv 3
Q.15 F = αa2ijˆ Q.16 (a) , (b) , (c) zero
4qa 4a

m µ0 I I'c  1 1 
Q.17 T0 =
2π 0.57s
= Q.18  −  to the left
6IB 2π  a b 

d I µ0 Qω
Q.19 (i) z=0, x = ± (ii) Q.20 h2 tan2 θ
3 2πd πλ 4

µ0  4I  µ0  I2  1
Q.21 (i)   along Y-axis,  (ii)   10 ,tan4   + π with positive axis
4π  a   
4 π  2a  3

Qvµ0 I  3 3   π 3  2ˆ
Q.22(a)  − 1    (b)
= τ BI  − a j
m 6a  π   
 3 4 
mg mg µ0iJ0  d
Q.23 (a) I = (b) I = Q.24 tan−1   −kˆ ( )
( ) πrBx π  2h 
1/2
πr B2x + B2y

Q.25 0.62N<F<0.88N = 4.7 × 103 T
Q.26 Bmin

µ0 I1 I2
Q.27 i 1 0.1110A,i
= = 2 0.096A Q.28 In (3) along –ve z direction

µ0 I2  a2 
Q.29 (b) Yes Q.30 F = In   , zero
2π  L2 + a2 

Exercise 2

Single Correct Choice Type


Q.1 C Q.2 A Q.3 C Q.4 C Q.5 C Q.6 D
2 1 . 6 8 | Moving Charges and Magnetism

Q.7 C Q.8 D Q.9 B Q.10 A Q.11 A Q.12 A


Q.13 A Q.14 C Q.15 C

Multiple Correct Choice Type


Q.16 B, C Q.17 A Q.18 B, D Q.19 A, B, C Q.20 A, B, C, D Q.21 A, B
Q.22 A, D Q.23 C, D

Assertion Reasoning Type


Q.24 D Q.25 B Q.26 D Q.27 C Q.28 D Q.29 D

Comprehension Type

Paragraph 1:
Q.30 A Q.31 A Q.32D
Paragraph 2:
Q.33D Q.34 B Q.35 B

Matric Match Type or Match the Column


Q.36 A → q, r; B → p; C → q, r; D → q, or; A → q, r; B → p; C → q, r; D → q, s
Q.37 A → p, r, s; B → r, s; C → p, q; D → r,s

Previous Years’ Questions


Q.1 C Q.2 C Q.3 B Q.4 A, B, D Q.5 A, B, D Q.6 A, C
Q.7 A, C Q.8 A, C, D Q.9 B, D Q.10 C, D Q.11 5 Q.12 B
Q.13 D Q.14 A, C Q.15 B Q.16 3 Q.17 C Q.18 B
Q.19 A, B, C Q.20 A, D Q.21 A, C

Solutions

µ0 × 90 µ0
JEE Main/Boards Sol 4: B =
3
=
π
× 30 = 1.2 × 10–5T,
2π ×
2
Exercise 1 towards south.

µ0NI µ0 × 100 × 0.4 1


Sol 1: B = = = 3.1 × 10–4T Sol 5: F =I  ×B = 8 × 1 × 0.15 × = 0.6 Nm–1
2R 2
2 × 8 × 10−2
µ0 I a
Sol 6: F = ·
2
µ0 I µ0 × 35 a a2
Sol 2: B = = = 3.5 × 10–5T 4 π x2 + x2 +
2πr 1 4 2
2π ×
5 Since force is always perpendicular to velocity so path
will be a circle
µ0 I µ0 × 50
Sol 3: B = k̂ = k̂ = 4 × 10–6T,
2πr 5 mv 9.1 × 10−31 × 4.8 × 106
2π × R= = = 4.2 cm
2 qB 1.6 × 10−19 × 6.5 × 10−4
vertically upward.
P hysi cs | 21.69

Sol 7: (i) τ = M × B; M = INA  20 


=iB×  
τ = 6 × 30 × π (0.08)2 ×1 × sin60º = 3.1 Nm  100 

(ii) No = 7 × 0.2 × 1.5= 2.1 N vertically downwards.

−µ0 × 20 × 16 µ0 × 25 × 18 (c)
Sol 8: B = +
16 10 10
2× 2× 6
100 100
= 5 π × 10–4T toward west 8 8

Sol 9: For tension to be zero


(a) F = I  ×B = mg Effective length of wire is 16 cmin the magnetic field so
2mKE 
= 5 ×0.45 × B = F = i  ×B
qB
0.6 = 7 × 0.16 ×1.5
B= = 0.26 T
5 × 0.45 = 1.68 N downwards

(b) By force equilibrium


Sol 12: Length of wire = N × 2 π R
2T = Mg + F N × 2πR
Final no. of turns = = 2N
= 0.06 × 9.8 + 0.06 × 9.8 = 1.176 N R

2
Sol 10: I = 300A; Force per unit length = F Magnetic moment µ = INA

µ0i1i2 µ0 × (300)2 µ1 N1 A1 N × πR 2 2
F= = = 1.2 Nm–1 = = =
2πd 3 µ2 N2 A2 2 1
2π × × 10−2 R 
2 2N × π  
Since the direction of the current is in opposite direction 2
in the wire, the force will be repulsive in nature. Sol 13: N = 20
r = 0.1 m
Sol 11: B = 1.5T; r = 0.1m B = 0.1 T
I=5A
B = 1.5 T 
(a) τ = M × B = MB sin0º = 0

7A (b) F = i  × B

F = Total force is zero as  is zero for a closed loop
 
 (c) Force on each electron = q v × B
(a) F = i  × B IB
= eVB = = 5 × 10–25 N
= 7 × 0.2 ×1.5= 2.1 N vertically downwards. nA

(b) Sol 14: Refer page 21.11 to 21.14

 Sol 15: Refer page 21.11 and 21.12

 Sol 16: B at the centre is B = µ0NI


F = i 1 × B
N - number of turns

= i 1Bsinθ I - current
2 1 . 7 0 | Moving Charges and Magnetism

mv Sol 25: Force per unit length


Sol 17: R =
qB
µ0i2
RqB F= = 6 × 10–7 N/m
v= 2πd
m
vp RqB md
= =2 Sol 26: Magnetic field inside the solenoid is
vd mp RqB
400
B = µ0NI = µ0 × ×3
0.4
Sol 18: Refer page 21.25 to 21.26 = 4 π ×10–7 ×3 ×1000 = 12 π × 10–4 T

 Torque on the coil is τ = M × B = MB
 
Sol 19: F = q E + q v × B
= 0.4 × 10 × π (0.01)2 × 12 π × 10–4 =5.9 ×10–6 Nm

Sol 20: Its path will be a circle.


Sol 27: Let the resistance of the voltmeter be R
Voltage across 300 Ω = 60 – 30 V= 30 V
Sol 21: Magnetic field due to side BC is BBC
30
µ I  3 I= = 0.1 A
3 3 3µ0 I 300
BBC = 0  +  ; BBC =

4 πR  2 
2  4 πR Let equivalent resistance of voltmeter and 400Ω be Req
Magnetic field due to all sides will be equal so IReq = 30V

3 3µ0 I 0.10 Req = 30 V


Bnet =
4 πR R × 400
Req = 300 Ω =
R + 400
Sol 22: Electron moving in a circle will act like a loop
carrying current I. 3R + 1200 = 4R
q qω qv Resistance of voltmeter = R = 1200 Ω
So,I = = =
t 2π 2πR When voltmeter is connected to 300 Ω
µ0 I µ0 qv
So magnetic field at centre = B = = 1200 × 300
2R 4 πR 2 Req = = 240 Ω
1500

9 × 109 × 1.6 × 10−19 × 2.2 × 106 60 6


Thus B = 14.1 Wb i= A= A
( ) 640 64
2
0.5 × 10−10
6
Voltage measured = ×240 = 22.5 V
64
Sol 23: B = µ0 ni
Sol 28: (i) i = 9.6A
5000
= 4 π × 10 ×–7
× 5 = 1.57 ×10–2T
2 2m
⊗ ⊙
Sol 24: B = 2.5 T 10
m
v = 1.5 × 10 m/s 7
11
  1
F = qv × B = q × 1.5 × 107× 2.5 × 10
2 B at m from wire B is
2.5 11
= 1.6 × 10–19 ×1.5 ×107 × × 3 × 10–12
2
µ0 × 9.6 µ0 I
B= − 0
=
 12  10
2π   2π ×
 11  11
P hysi cs | 21.71

9.6 × 10 µ0
I= = 8A = µ0 × 1250= × 5000π = 5π × 10−4 T
12 4π
µ0i1i2
(ii) Force per unit length F = Sol 6: (A)
2π × 2 y
µ0 × 9.6 × 8
= = 7.68 × 10–6 Nm-1
4π a

x
Exercise 2
z
Sol 1: (D) Total magnetic field at point O
µ0i ˆ µ0i ˆ
Magnetic field = B = – i+ j
µ0  3 µ0  1 2πa 2πa
=B +
2R ' 4 2R 4

µ   3 1 Sol 7: (B) E = −K1ˆj ;
= 0  + 
8 R ' R  K1 is some constant

V = K 2ˆi
Sol 2: (C)
  
F = q V × B + qE = 0
    
⇒ V × B = − E ⇒ B = −k̂
R

Sol 8: (D) Final velocity of the particle

2
µ0 I  θ  ˆ  qEt 
Magnetic field B =   ( −k) =v= v 20 +  = 2v0
2R  2π   m 

  2
 qEt 
 µ0 q V × r µ qV sin θ v 20 + 2
 = 4 v0
Sol 3: (A) B = = 0  m 
4π r3 4π r2
2
µ0 2 × 100 × sin30º  qEt  2 3 mv 0
= × = 10−7 × 25 =2.5mT   = 3v 0 ⇒ t =
4π 4  m  qE

Sol 4: (A) Magnetic field at the common centre is Sol 9: (D)

µ0 I ˆ µ0 I ˆ µ0 I ˆ
i+ j+ k •
2R 2R 2R qE
µ0 I
= 3
2R V0

When electric field is applied


Sol 5: (A) Magnitude of magnetic field at the centre
mv 20
= qE
µ0 × 20 × 16 µ0 × 25 × 18 R1
=+ −
2 × 16 × 10 −2 2 × 10 × 10−2 mv 20
R1 =
qE
3
= – µ0 × 10 + µ0 × 2250
2 1 . 7 2 | Moving Charges and Magnetism

When magnetic field is applied mv 2mKE


Sol 13: (C) R = =
mv 0 qB qB
R2 =
qB
2m(2KE) 2
R’ = =R
R1 mv 20 qB v0 B q(3B) 9
= =
R2 qEmv 0 E
Sol 14: (C)
Sol 10: (B) Vy = 4
B0

Vx = 5
(d, 0, 0) B =10
V
Y-component of velocity will make the particle to move
in circle whereas x-component of velocity will make
particle move along x-axis.
For the particle to not hit y-z plane radius of the particle So motion is helical.
should be less than equal to d

mv Sol 15: (A) Force on a particle moving in magnetic field


R= ≤d  
qB0 is qv × B .
 
qB0 d (4iˆ + 3ˆj) × 10−13 =1.6 × 10–19× 2.5 ×107 (K × B)
vmax =
m
Force will be zero if direction of magnetic field and
velocity is same.
kq1q2 kq2 
Sol 11: (A) Electric force Fe= = So B = (0.6 î × –0.8 ˆj ) B
r2 r2
µ qv  ⇒ (4 î + 3 ˆj ) = 1.6 × 25 ( k̂ × (0.6 î – 0.8 ˆj ) B
Magnetic force = qv  0 ×  
2 
 4π r  ⇒ B = –0.075 î + 0.1 ˆj
 µ0  1
Fm= q2v2  4 π  2  
  r Sol 16: (A) Force acting on particle = q. v × B

Fe k 1 c2 ⇒ q.2 î × B = –2 ˆj
= = = 
Fm µ  v 2 ε0 µ0 v2 ⇒ B is in +ve z direction ( k̂ )
v 2  0 
 4π  Electric force on the particle is zero.

mv 2mKE So when v3 =2 k̂ , force is zero.


Sol 12: (B) R = =
qB qB
Sol 17: (A) Magnetic field is in (– k̂ ) direction
2KE × 1 × mp So direction of force
R = (mp = mass of proton)   
H+ eB F = q v ×B

2KE × 4 × mp F̂ = –[– î ×(– k̂ )]= ˆj


R =
He+ eB
F
Sol 18: (C)
2KE × 16 × mp
R =
O+2 2eB

So, R =R mgcos 
He+ O +2
mgsin 
P hysi cs | 21.73

F = qVB 2π 2πr
t= =
Particle will leave the inclined plane when ω v

F = mgcos θ ⇒ qvB = mgcos θ q.πr 2 v qrv


=M =
2πr 2
mgcos θ
v=
qB

Time taken to reach v is t Previous Years’ Questions


v = gsin θ t
v mgcot θ mcot θ Sol 1: (D) Net magnetic field due to both the wires will
t= = =
gsinθ qgB qB be downward as shown in the figure.


Sol 19: (D) y v
⊗ z
y
×
I –I

x
x
T B
T d
→ →
Since, angle between v and B is 180°.
 
F = I ∫ d × B
Therefore, magnetic force
= I (2r î )× (–0.2 k̂ ) = 20 ˆj → → →
Fm = q ( v × B ) = 0
Magnetic force is in +ve y direction
So balancing force on semi-circular ring we get Sol 2: (C) H1 = Magnetic field at M Due to PQ +
magnetic field at M due to QR
2T = 20 ⇒ T = 10N
But magnetic field at M due to QR = 0
Sol 20: (B) ∴ Magnetic field at M due to PQ (or due to current I
 in PQ)= H1
 B
B Now H2 = Magnetic field at M due to PQ
(current I) + magnetic field at M due to QS (current I/2)
+ magnetic field at M due to QR
H1 3 H 2
B =H1 + +0= H1 ; 1 =
2 2 H2 3
 Note: Magnetic field at any point lying on the current
B carrying straight conductor is zero.
Torque due to magnetic field will be balanced by gravity.
B=0
mgsin θ R = I× π R × B sin θ
2

mg
B=
πiR
i
Sol 21: (B) Magnetic field = I × A

q.πr 2
M=
t
2 1 . 7 4 | Moving Charges and Magnetism

Sol 3: (B) If the current flows out of the paper, the Note: This is a common practice, when by assuming
magnetic field at points to the right of the wire will be equal currents in opposite directions in an imaginary
upwards and to the left will be downwards as shown in wire (here AB) loops are completed and solution
figure. becomes easy.


B Sol 5: (C) Consider an element of thickness dr at a
i distance r from the centre. The number of turns in this
  N 
B element,dN =   dr
 b – a
B Magnetic field due to this element at the centre of the
coil will be
Now, let us come to the problem.
µ0 (dN)I µ0 IN dr
Magnetic field at C = 0 dB = = .
2r 2 b–a r
Magnetic field in region BX’ will be upwards (+ve) r =b
because all points lying in this region are to the right of µ0NI b
both the wires.
∴B= ∫ dB = 2(b – a)
ln  
a
r =a

X X’
C B
A
b
Magnetic field in region AC will be upwards dr
(+ve),because points are closer to A, compared to B. r
Similarly magnetic field in region BC will be downwards
a
(–ve).
Graph (B) satisfies all these conditions. Therefore,
correct answer is (B).

Sol 4: (D) The magnetic field at P(a, 0, a) due to the Note: The idea of this question is taken from question
loop is equal to the vector sum of the magnetic fields number 3.245 of IE Irodov.
produced by loops ABCDA and AFEBA as shown in the
figure. mv
Sol 6: (B) Radius of the circle =
Bq
C
or radius ∝ mv if B and q are same.
P(a, 0, a)
(Radius)A> (Radius)B; ∴ mAvA> mBvB


D B î Sol 7: (A) Magnetic field at P is B , perpendicular to OP
E
in the direction shown in figure.
ˆj
y
P(x, y)
F r
A
 
Magnetic field due to loop ABCDA will be along î and i  B
× x
due to loop AFE BA, along k̂ . Magnitude of magnetic o
field due to both the loops will be equal. Therefore,
1
direction of resultant magnetic field at P will be
2
( î + k̂ ).
P hysi cs | 21.75

→ Sol 11: (D) Magnetic force does not change the speed
So, B = B sin θ î – B cos θ ˆj of charged particle. Hence, v = u. Further magnetic field
on the electron in the given condition is along negative
µ0 I
Here, B = y-axis in the starting. Or it describes a circular path in
2πr clockwise direction. Hence, when it exits from the field,
y x y < 0.
sin θ = and cos θ =
r r Therefore, the correct option is (D)
→ µ0 I 1 µ I(yiˆ – xj)
ˆ
∴B = . (y î – x ˆj ) = 0 → → →
2π r 2
2π(x + y 2 )
2
Sol 12: (A) F m = q ( v × B )
(as r2 = x2 + y2) ∴ Correct option is (A)

Sol 8: (B) If (b – a) ≥ r Sol 13: (C) Correct answer is (C), because induced
electric field lines (produced by change in magnetic
(r = radius of circular path of particle)
field) and magnetic field lines form closed loops.
The particle cannot enter the region x > b.
So, to enter in the region x > b Sol 14: (A) If we take a small strip of dr at distance r
from centre, then number of turns in this strip would
mv q(b – a)B
r > (b – a)or > (b – a)or v >  N 
Bq m be, dN =   dr
b – a
Sol 9: (B) Electric field can deviate the path of the Magnetic field due to this element at the centre of the
particle in the shown direction only when →it is along coil will be
negative y-direction. In the given options E is either
µ0 (dN)I µ0NI dr
zero or along x-direction. Hence, it is the magnetic field dB = =
which is really responsible for its curved path. Options 2r (b – a) r
(a) and (c) cannot be accepted as the path will be r =b
µ0NI b
circular in that case. Option (d) is wrong because in that ∴B= ∫ dB = ln
n  
case component of net force on the particle also comes r =a 2(b – a) a
in k̂ direction which is not acceptable as the particle is
moving in x-y plane. Only in option (b) the particle can
Sol 15: (B)
move in x-y plane.
→ → → → 2mK
In option (d) F net = q E + q ( v × B ) r= ⇒r∝ m
Bq q
Initial velocity is along x-direction. So, let rα= rp < rd

v =v î

F net = qa î + q [(v î ) × (c k̂ + b ˆj )]
Sol 16: (A) Bnet = BM + BM + BH
1 2
= qa î – qvc ˆj + qvb k̂

µ0M1 µ0M2
= + + BH
In option (b) F net = q (a î ) + q[(v î ) × 4 πx 3
4 πx3

(c k̂ + a î )]= qa î – qvc ˆj µ0
= (M1 + M2 ) + BH
4 πx3
→ →→
Sol 10: (C) U = – MB = – MB cos q 10−7
→ = × 2.2 + 3.6 × 10−5
Here, M = magnetic moment of the loop 10 −3
→ →
θ = angle between M and B
= 2.56 × 10−4 Wb / m2
U is maximum when θ = 180° and minimum when θ = 0°.
So, as θ decreases from 180° to 0° its PE also decrease.
2 1 . 7 6 | Moving Charges and Magnetism

Sol 17: (A) µ0H =µ0ni µ0 × 10


= 10–6 î +
1
3 100 2π ×
3 × 10= × i ⇒=
i 3A 2
0.1
= 10–6 î + 4 × 10–7 × 10 î

Sol 18: (B) Since B is uniform,
 only torque acts on a
current carrying loop. =

τ (IA) × B = 5 ×10–6 î T
   
A = Ak for (b) and A = −Ak for (d).
Sol 3: Magnetic field can be found as the super position

∴ τ =0 for both these cases. of both given below.
  
The energy of the loop in the B field is: U = −IA ⋅ B ,
which is minimum for (b).

I
JEE Advanced /Boards y 45º 45º
a/2
Exercise 1
x
Sol 1: I1 = I2 = I3 = I4
z
⇒ F1 = F2 = F3 = F4 = F a
⇒ 2F Magnetic field due to loop = Bl

 
2F  
µ0 I  1 1 
= – 
  a   2
+   × 4kˆ
45º 2 
2 F  4π   
2
45º    2 

2F I1 I2
Resultant force will be 2 2 F from right to left 1 ⊗ F2+F4
⊗2
45º
Sol 2: Let magnetic field due to wire be Bw Fres
45º
(a)x = 0, z = 2m; F1+F3
µ0 I ˆi ⊙ ⊙3
B = B 0 + Bw = – + 10–6 î 4
2π × 2
µ0 I
= –10–7 ×10 î + 10–6 î =– × 2 ×4 k̂
2πa
=0
2 2µ0 I
(b)x = 2m, z = 0 = − k̂
πa
µ0 I
B = B 0 + Bw = k̂ + 10–6 î µ0 Ikˆ
2π × 2 Magnetic field due to infinite length wire = Bw =
a
µ I 2π  
B = 10–6 k̂ + 10–6 î = 2 ×10–6 T = 0 k̂ 2
πa
(c)x=0, z=–0.5m (1 − 2 2)µ0 I
Net magnetic field = k̂
πa
B= B0 + Bw
P hysi cs | 21.77

Sol 4: Sol 5: Magnetic Induction

y  µ I 1 µ I1 µ I
B = 0   kˆ + 0   kˆ + 0 ˆj
2 2(2R)  4  2R  4  4 πR
45º 1
x µ0 I  3 ˆ 1 ˆ 
1amp 1 =  k + j
4R  4 π 
z
i2
Sol 6: Magnetic Induction
π  π
1  µ0 I  3 2  µ I
i1 = 2 × 1 = amp B=   kˆ + 0 kˆ
2π 4 2R  2π  4 πR
 
 
1 3 µ0 I 3 ˆ µ0 I ˆ µ I  3π 
i2 = 1 – = amp = × k+ k = 0  + 1 kˆ
4 4 2R 4 4 πR 4 πR  2 
1
µ0    3π 
4  
Magnetic field due to i1 = B1 =–    2  k̂ Sol 7: Magnetic Induction
2 2  2π 
 
µ0    µ I µ I µ I
3
= − × k̂ B = 0 ˆi – 0 ˆi – 0 kˆ
8 2 4 2R 4 πR 4 πR
3 π
µ0    
 4   2  k̂ µ0 I µ0 I ˆ
Magnetic field due to i2 = B2 = = [2π − 1] ˆi – k
2 2 2π 4 πR 4 πR

3µ0 1 µ0 I
= × k̂ =  4 π2 + 1 − 4 π + 1 
4 πR  
8 2 4

µ0 I
Magnetic field due to wire in x-direction = B3 =
4 πR
(
2 2π2 − 2π + 1 )
µ0 × 1
B3 = (sin(–45º ) + sin90º ) kˆ
4π × 1 Sol 8: We will find magnetic field B by ampere’s law.
 
µ0  1 ˆ �∫ B.dl = µ0 IIN
B3 = 1 − k
4π  2
r
1 1m
1 2
1 (a)For r1< R
1amp
Magnetic field due to wire in negative y-direction =By B×2 π r1 = µ0 ( ∫ JdA )
 µ ×1   r1 
By = –  0 (sin(–45º ) + sin90º )  kˆ =μ0  br2πr dr 
 4π × 1   ∫0 
 
µ0  1 ˆ 2πbr13
= − 1 − k B×2 π r1 = µ0
4π  2 3
Net magnetic field = B = B1 + B2 + B3 + B4= 0 µ0br12
B=
3
2 1 . 7 8 | Moving Charges and Magnetism

0 Sol 11:
y
(b)

By ampere’s law
 r0 
B×2 π r2 = µ0 ∫ (JdA) = µ0  ∫ br2πr dr  x
0  q, m V
 
r03
B×2 π r2 = µ0 2πb
3
y coordinate is equal to twice the radius of the circle
µ0br03
B= y = 2R
3r2
mV0 2mV0
R= ⇒y=
qB qB
Sol 9:

i Sol 12: We know that velocity of charged particle = v


E
a =
B
mv
Force = Change in momentum per sec=
t
e mE I
I= ⇒F=
µ I t B·e
Magnetic Force = qVB = qV  0 
 2πa 
Sol 13: Force acting on a wire carrying current
 
Sol 10: F = I∫ d  × B

y Since B is uniform so
E= 5× 10
7  
F = I ( ∫ d ) × B

For a loop ∫ d  = 0
x
So F = 0
6
V= 5× 10 m/s z
Sol 14:
Magnetic force = qVB
Electric force = qE


j

When both forces are equal in magnitude and opposite xB
in direction then net force on charged particle is zero.
qVB = qE


R R i
I
E 5 × 107
B= = = 10 T


k
V 5 × 106 


 = 2Ri
and direction is in positive k̂ direction
 
Force = I ∫ d  × B

Since B is constant so
 
F = I ( ∫ d ) × B
 
F = I ·  ×B
P hysi cs | 21.79

Consider a loop PQRS placed in uniform magnetic


F= I ( 2Riˆ × ( −Bk)
ˆ
) field B in such a way that the normal to coil subtends
an angle θ to the direction of B when a current I flows
= I 2RBjˆ = 2IRBjˆ
through the loop clockwise.
The sides PQ and RS are perpendicular to the field
Sol 15: F = F1 + F2 + F3 + F4 and equal and opposite forces of magnitude I and B
= i∫(d 1 ×B1)+ i∫ (d  2×B2) + i∫ (d  3×B3) + act upwards and downwards respectively. Equal and
opposite forces act on sides QR and PS towards right
i∫ (d  4×B4) and left of coil.
 a  a
The resultant force is zero but resultant torque is not
=  i ∫ dyjˆ × (αy)( −k)
ˆ  + i(aiˆ × αy( −k))
ˆ + i dyjˆ × (αy)kˆ + I × 0
∫ zero. The forces on sides PQ and RS produce a torque
 
 0  0
due to a single turn which is given by
a2 ˆ αa2 ˆ τ = I2B sin θ
F1 = −i α i + i αa2ˆj + i i = i αa2ˆj
2 2
for small θ , sin θ ≈ θ
τ = I2Bθ  ... (i)
Sol 16: τ = Iα
B
xxxx
 m 2 m 2 
xxxx =  ×2 + × 2α
E î  4 12 4 4 
xxxx  

 1 1 m2  4  m2
a = m2  +  α =  =  ... (ii)
 24 8  8 3 6
2a By (i) and (ii)
2v
m2
I 2 Bθ = α
(a) Work done by Electric Field = Change in Kinetic 6
Energy 6 IB
α = θ
1 1 m
∫F.dx = m(2v)2 − m v 2
2 2 6 IB
ω2 =
3 m
qE × 2a= mv 2
2 m 10−2
Time period = 2π = 2π
3mv 2 6 IB 6 × 2 × 10−1
E=
4qa 1
=
2π = 0.57 sec
3 120
(b) Rate of work done = F.v = qE.v= mv 3
4a
(c) Work done by magnetic field is always zero. Sol 18: Net force acting on the loop = F

Work done by electric field = F.vˆ = qE î . (–2v ˆj )= 0 µ0 I I'c µ0 I I'c µ0 I I'c  1 1 
F= − =  − 
2πa 2πb 2π  a b 
Sol 17:
O This loop will experience attractive forces.

P 

Q B
i

S
O1 R


2 1 . 8 0 | Moving Charges and Magnetism

Sol 19: (i) z


cos θ =
d + z2
2
y
µ0i2 
F=
2π d2 + z 2

Resultant force is downward


i
−2µ0i2  z
Fnet = –2 Fcos θ = ·
x 2π d2 + z 2 d2 + z 2
d
−µ0i2 z
d Fnet =
π (d2 + z 2 )

For small z
Net Force at some point x, y is

µ0 I µ0 I µ0 I −µ0i2 z
Fnet = + + =0 Fnet = = λa
2π(x + d) 2πx 2π(x − d) π d2

1 1 1 µ0i2 
⇒ + + =0 ω =
n+d x x−d λπd2
2x 1 µ0i2 µ0
+ =0 ω 1 i
2
x −d 2 x F= = =
2π 2π λπd 2 2πd λπ

2x2 + x2 − d2
= 0 ⇒ 3x2 = d2
x(x2 − d2 ) Sol 20: l cos θ = h

d

x= ±
3
Net force will be zero only in x–y plane
y
d θ
i.e. when z = 0 and x = ±
3  h
(ii) z

⊗ z
h tan 
⊗ ⊗ x Take a ring at distance y from the top point of the cone.
Let the middle wire is displaced by z distance in positive Magnetic moment M = IA
z-direction. dM=
Attractive force acting on wire is F  
 Q 2πydy tan θ  ω
 ·  · π (y tan θ)2
 π(htan θ) h cos θ  2π
F F  
 cos θ 
θ z h
Qω tan3 θ Qω tan2 θ h4
= ∫ · y 3dy = ·
0 h2 tan θ h2 4
d
P hysi cs | 21.81

1 Sol 22: (a)


= Q ω tan2 θ h2 v
4 R

Sol 21: (i)BC= BA = BB = BD = B 60º 60º

µ0 I × 2 R
B=
2π 2 a 2
y
BD + BB
1 3 ˆ
⊙ 45º ⊗ 
v = v  ˆi + j
BC + BA 2 2 
x 
  2π  
⊙ ⊗ µ I  µ I  3 3  
B =  0  3 − 0  +   k̂
 2R  2π  4 πR  2 2  
   
   2 
Net magnetic field is µ I 3µ0 I  µ I 1 3
µ I ×2 B=  0 −  k̂ = 0  −  k̂
Bnet = B 2 = 0 along y-axis  6R 2πR  2R  3 π 
2π a  

qv  ˆ µ I1 3  ˆ
(ii)
F2 Force = qv×B =
2 
( )
 i + 3 ˆj × 0  − k
2R  3 π  

 
D A
F1 Qv µ0 I  3 3 
=  − 1
F1 m 6a  π 
 
(b)Net Torque = M × B = I AB ˆj
C B
µ0 I2  π 3a2  ˆ
F1 = = I  a2 − B j
2π(2a) 3 4 

µ0 I2  π 3 2 ˆ
F2 = = B I − a j
3 4 
2π(2 2a)  

F2 µ0 I2 λ  1 3 µ0 I2 λ
F x = F1 + = 1 +  = 8πa Sol 23: (a) Net Torque on the loop is
2 4 πa  2
τ = – MBx ˆj + MBy î = Ιπr 2 B2x + B2y  ...(i)
2 2
F2 µ I λ 1 µ0 I λ
F y = F1 – = 0 1 −  = 8πa
2 4 πa  2 By Torque balance mgr = τ  ...(ii)
By (i) and (ii)
µ0 I2  2
Net force = 1+3 ;  =1 mg
8πa I=
πr B2x + B2y
 µ0   I2 
=
 4 π   2a  10 (b) Net Torque is τ = –MBx ˆj
  
Radius = R = a | τ | = Iπr 2Bx
By torque balance
mgr = t ⇒ mgr = Iπr2 Bx
mg
I=
πrBx
2 1 . 8 2 | Moving Charges and Magnetism

Sol 24: Magnetic field due to sheet of width d and Sol 26:
infinite length at a distance h is given by S
d  0.1m
µ j
B = 0 0 tan–1  2  ˆi
π  h 
  v
  
 = ˆj 2
 
F = i  ×B
G v
iµ 0 j 0  d ˆ
F= tan−1   (–k)
π  2n 
v 3

Sol 25: 2
R
Electron will move in helical path with pitch = 0.1 m. For
minimum value of B particle should reach at point S in
a single revolution.
I
2πm
Time period T =
qB
v
So 0.1 = T
2
 
Force= I  × B = 10 × 0.5 × 0.1 v ·2πm
0.1 =
1 2·qB
Force = N upward on inclined plane
2
20πmv
B’ =
2q

20π 2 × 9.1 × 10−31 × 2000 × 1.6 × 10−19


3 3 =
 2 × 1.6 × 10−19

 
2 2
1
mg 
2 2 × 9.1 × 10−31 × 2000
1 = 10π
2 mg  N 1.6 × 10−19
2
 mgcos  B = 10π 2.275 × 10−4

B = 4.7 × 10–3 T
F
Sol 27: To neutralize the magnetic field, current in
3 vertical ring should be such that the magnitude of
3 3 magnetic field is 3.49×10–5 T and current in horizontal
4
4 ring should be such that the magnitude of magnetic
3 3 3 3 1
µ .1 + F = ⇒F= – field is × 3.49 × 10−5
4 4 40 3

3  3 For vertical ring


Fmin = 1 −  = 0.62 N
4  10 
µ0NI µ0 × 100 × I
B= =
3 3 3 2r 2 × 0.2
F= +
4 40
3.49×10–5 = µ0 × 250 I
Fmax = 0.88 N
P hysi cs | 21.83

3.49 × 10−5 3.49 × 10−5 Sol 29: (a) |B1| = |B2| = |B3| = |B4|
I= = = 0.111 A
µ0 × 250 4 π × 10−7 × 250

For horizontal ring x


2
µ0NI 1 −5 µ0 × 100 I
B= ⇒ × 3.49 × 10 =
2r 3 2 × 0.3 i
1 3 a
⇒ I = 0.096 A
4
Sol 28:
2
i1 a
x 
2
y
4
z y R
dB
 B1  B3
dy B13
 P
30º R y
x
60º (a, 0 , 0) ⊙  x
dB (a, 0 , 0) 
Qa
2
Q
y
P 2
a
x 
2
dB 2
 a z
dy 2 y
2 a
x 
2
 4
R y R
x
0) ⊙  x a x
dB (a, 0 , 0)
2 2
a
x 
2
2
Q
Force on dy element in x direction is  
 
= ∫ i2dy B sinθ µ0 I a a
∫ dF B1 = 

+ 

a2  a2 a2 
4 π x2 + 2 x2 + 2 x2 +
Rdθ µ0i1 4  2 2 
F = ∫ i2 · sin θ
cos θ 2πR
µ0 I a
B1 = ·
30 2
µ0i1i2 a a2
= ∫ tan θ dθ 4 π x2 + x2 +
2π −60
4 2

µi1i2 Resultant of B1 and B3 is B13 = 2B1cos θ


F= [log cos]30
−60

a
µi1i2 µi1i2 2 × µ0 I a 2
= = log 3 log3 ·
2π 4π B13 =
a2 2 a2
2 a2
4π x + x + x2 +
4 2 4
2 1 . 8 4 | Moving Charges and Magnetism

µ0 Ia2 µ0 I2 α
µ0 I2 α
= = ∫ cot θ dθ = ln(sin θ)90
1 π π
π
 a2   a2  2 2
4 π  x2 +   x2 + 
 4   2 
 µ0 I2  
a
= ln  − 1
µ0 I a2 2π  L2 + a2 

B13 =
1
 a2  2 If direction of current in B is reversed then resultant
π(4x2 + a2 )  x2 + 
 2  magnetic field will become horizontal and so net force

will be zero.
µ0 Ia2
Similarly B24 =
1
 a2  2
π(4x + a )  x2 + 
2 2

 2  Exercise 2
Net resultant = B13 + B24 Single Correct Choice Type

2µ0 Ia2 Sol 1: (C) Magnetic field at some x is given by


=
1
 a2  2
π(4x2 + a2 )  x2 + 
 2 
 4I
(b) Yes

Sol 30: Bres = 2Bcos θ (–d, 0, 0) (d, 0, 0)



Fres = I ∫ d  × Bres
I

R dθ 2 µ 0 I a µ0 (4 I) ˆ µ0 I ˆ
= ∫ I × sin θ cos θ ; sin α = k+ k
2πR L2 + a2 2π(d − x) 2π(d + x)

µ0 I  4 1  µ0 I  5d + 3x 
=  +  =  
2π  d − x d + x  2π  d2 − x2 

It corresponds to graph (c)

x Sol 2: (A) Magnetic field at the centre due to Rdθ


component is
B
 
R dx B 
 By 
⊗  d
R B Bx Rd
L
  I 
µ0  ·Rdθ 
a
Bx = ∫ dBx = ∫ 
2πR  cos θ = µ0 I
2πR 4 π2R

 I 
µ0  ·Rdθ 
By = ∫ dB y = ∫ 
2πR  sin θ = µ0 I
2πR 4 π2R
P hysi cs | 21.85

µ0 I Distance from the point of projection = tv cosθ


B= 1+1
2
4π R 4 πM 4 πMv cos θ
= v cosθ =
QB QB
E
Sol 3: (C) V = for no deflection to occur
B
3.2 × 105 Sol 7: (C)
V= = 1.6 × 108 m/s
2 × 10 −3

mv 9.1 × 10−31 × 1.6 × 108


R= = = 0.45 m 
qB 1.6 × 10−19 × 2 × 10−3
V 
  2
q
Sol 4: (C) = α V 
m
1 
Work done by electric field = qE0x0= m(25–0)
2 R
25m 25
x0 = =
2qE0 2αE0  π + 2θ   π + 2θ 
Time taken =  =  T
 ω   2π 
Sol 5: (C) Particle is moving in helix along y-axis. So
the time taken by particle to reach in x-z plane should Sol 8: (D)
be integral multiple of time taken to complete one
revolution.
V
y



  2
x 


V
z  π − 2θ   π − 2θ 
Time taken =   =  T
Helical motion of the particle  ω   2π 
2mv  2πm 
⇒ =  n
qE  qB  Sol 9: (B)

Bv
n= (2, 2)
πE

 Bv 
So   should be an integer 
 πE  

Sol 6: (D) Both particles will move in helix. They will


meet for the first time when mass m will complete two
(2, –2)
revolutions and mass 2m will complete one revolution.
Time taken to complete one rotation. Magnetic force is given by
2 × 2πM 2π2M  
=t1 = ; t2 dFm = i ∫d  × B = i∫d (– ˆj ) × (–4 k̂ ) = 4i∫d  î
QB QB
2 1 . 8 6 | Moving Charges and Magnetism

since  and B are perpendicular so τ = MBsin θ

dfm = 8∫dl î = 8 × 4 î = 32 î τ = I × π R2 × B = I α

MR 2 α
Sol 10: (A) I π R2 × B =
2
y 2 × 4 × π × 10
α =
2
= 40 π rad/sec2
d
I
Sol 13: (A) Let us assume that resistance of p material

x is ρ and that of Q is q.
(–R,0,0) (R,0,0)
2ρ + q 2q + ρ
i1 = i , i2 = i
3(ρ + q) 3(ρ + q)

i1 2ρ + q
z =
i2 2q + ρ
B0
F = I∫Rdθ(sin θ î + cos θ ˆj ) × (–R cos θ ) k̂ I Q Y
2R
IB0R π Q
=
2 ∫ (sin θ ˆi + cos θ ˆj) × (− cos θ k)
ˆ dθ P
0
π i1
= ∫ (sin θ cos θ ˆj – cos2 θ ˆi) dθ P Q
0 i
X P i2 II
sin2θ ˆj (1 + cos2θ)iˆ 
π
= ∫ –  dθ
 2 2 We know that B ∝ i
0 
π π π SoB1 = magnetic field due to I part
 − cos2θ  ˆ  θ  ˆ  sin2θ  ˆ
=   j −   i −  i B2 = magnetic field due to II part
 4 0  2 0  4 0
For the magnetic field to be zero B1 = –B2 should hold.
π π
=0– î − 0 = – î Bi i1 2ρ + q
2 2 But ∝ = ≠ –1
Bi i2 2q + ρ

Sol 11: (A) Refer Q.18 Exercise-I JEE Advanced. So magnetic field will not be zero at centre.In (B), (C)
and (D) i1 = i2 so magnetic field is zero at centre.
Sol 12: (A) Torque on the ring due to magnetic field is
Sol 14: (C)
i I i
i1

B
i2
II

In (A)
3 i
i1 = i ; i2 =
4 4
P hysi cs | 21.87

−µ0i1 3µ0i2 Sol 15: (A) So its x coordinate cannot be positive.


B= (sin θ1 + sin θ2 ) + (sin θ1 + sin θ2 )
l 
4π 4π z B
2 2
v sin 
(B) V
i
i1 
y
v cos 
B
x
i2
i
z

By symmetry i1 = i2and magnetic field will be cancelled


out by both the parts.
(C) i1 y

45º 45º
x
L i2 Its x- and z- coordinate will be zero when particle will
complete one revolution.
y - Coordinate = vcos α t

3 i
i1 = i ; i2 = Multiple Correct Choice Type
4 4

Let magnetic field due to sides of square be Bs Sol 16: (B, C) (A)Motion is helical in nature
3 i
−µ0 i1   3µ0  (B) They will follow circular path with radius
Bs = 4 1 1 4 1 + 1 
 +  k̂ +   2mKE
L  2 2 L
4π 4π  2 2 R=
2 2 qB
Bs = 0 (C) Work done by magnetic force is always zero.

But magnetic field due to 2 infinitely long wires is not


(D)
zero so net magnetic field is zero.
i B
(D) x x x x x x x x x
i1 x x x x x x x x x
x x x x x x x x x
V
x x x x x x x x x
i
x x x x x x x x x
V
i i2 x x x x x x x x x
i x x x x x x x x x
By symmetry i1 = i2 =
2
So magnetic field due to four sides of square will cancel
out. Magnetic field due to two infinitely long wires will
also cancel out as they are equal in magnitude and
opposite in direction.
µ0 I
So net magnetic field is zero. Sol 17: (A, B, C) B = (sin θ1 + sin θ2 )
2πr
2 1 . 8 8 | Moving Charges and Magnetism

Sol 20: (A, B, C, D) On x-axis

i µ 0I µ 0I
(A) B = – =0
2πa 2πa

(B) On y-axis say at (y, 0, 0)

−µ 0 I µ 0I
B= kˆ + kˆ
2π(a + y) 2π(a − y)

So except at origin, B has only z-components

By ampere’s law magnetic field on a ring with centre as z


wire is same. (C) B1
1
B ∝
/ as θ1 and θ2 are also dependent on r. Bnet
r

B2
Sol 18: (B, D) Magnetic field 1
y y
O
2
A(0,1,0) x

(0,1,1)B D(1,1,1)
x (D) B cannot has x-component as B is perpendicular to
direction of I.
C(1,0,1)
z
Sol 21: (A, B) This can be done by applying magnetic
field in y-axis or z-axis.
µ0 I µ0 I
at A = BA = ;BB = y y
2π × 1 2π 2

µ0 I µ0 I
BC = ;BD =
2π × 1 2π × 2
x
x
1
Sol 19: (A, B, C) = c2
µ0 ε0 z z

Sodimension of y is m/s
Sol 22: (A, D)
V
E
v = when E and B are both perpendicular and
B
perpendicular to velocity 30º
B
So dimension of x m/s V
Dimension of RC = sec 60º

So Z = has dimension m/s
CR
2πm T1
So x, y, z have same dimensions. Time period T = ; a= =1
qB T2
P hysi cs | 21.89

mv sin θ Consider a point P in space between two wires at a


radiiR =
qB distance r from one wire. The magnetic force due to
wire 1 is in positive z-axis direction whereas due to wire
R1 sin(30º ) 1 2 is in negative z-axis direction.
= =
R2 sin(60º ) 3
Sol 28: (D) Statement 1 is false as Ampere’s circuital
pitch = v cos α t law holds good for a closed path of any size and shape
around a current carrying conductor only if the relation
P1 v cos(30º )
= = 3 is independent of distance.
P2 v cos(60º )

abc = 1; a = bc Sol 29: (D) Since angular acceleration of the mass will
not change so time period will also remain the same.

Sol 23: (C, D) If velocity is zero, then magnetic force is


zero.Energy cannot increase in magnetic field as work 
done by magnetic force is zero.
 
F = q v × B ;So force is perpendicular to its velocity. F =q

Assertion Reasoning Type



mgsin  mgcos 
Sol 24: (D) If initially velocity of charged particle is in mg
the direction of magnetic field then force acting on it
is zero and particle will continue to move in the same
direction. So statement 1 is false. Comprehension Type

Paragraph 1
Sol 25: (B) Magnetic field at any point is in tangential
direction. So it is not possible for a particle to move in Sol 30: (A) Magnetic field due to curved part is
tangential direction by the action of magnetic force.
µ 0 I  2π  µ 0 I
B=  =
4 πa  3  6a

B Sol 31: (A)

60º
Sol 26: (D) It’s velocity vector must be perpendicular to
both magnetic field and electric field.
60º

 
Sol 27: (C) F = I ∫ d  × B
So force acting is attractive

µ0 I 3µ0 I
B= (sin60º + sin60º ) =
1 2 a 2πa

i1 i2 2

d Sol 32: (D) Net magnetic field at C is


•P
r µ0 I 3µ0 I
B=– +
6a 2πa
2 1 . 9 0 | Moving Charges and Magnetism

Paragraph 1 3Kq 3Kq


Electric potential = − =0
r r
Sol 33: (D) I = 3A Magnetic field is zero as current due to rotating charge
r = 0.04m is zero.
+ –
N = 20
F
B = 0.5 T
r
Dipole moment M = INA= 3× 20 × π (0.01)2 –
F
+
M

= 1.88 × 10 Am
–2 2

1 F
Sol 34: (B) PE = –1.88 ×10–2 × + –
2
= –9.4mJ Magnetic moment= INA= 0 × NA = 0

Sol 35: (B) Torque, τ =in AB sin 90o E=



Kq

Kq
+
Kq
+
Kq

Kq

Kq
≠0
2 2 2 2 2 2
2  5a   5a   3a   3a  a a
 1             
= 3 × 20 × π ×   × 0.5 × 1 2 2  2  2 2 2
 100 
P
3 × 3.14 × 10−3 Nm
= = 9.4 × 10 −3 Nm

– + – + – +
Match the Columns
a M

Sol 36: A → q, r; B → p; C → q, r; D → q, or; A → q, r;


B → p; C → q, r; D → q, s Q
τ = MB sin 90º= 9.4 ×10–3 Nm Kq Kq Kq Kq Kq Kq
V= − + − + − =0
5a 5a 3a 3a a a
(A) Magnetic field is in opposite direction. Since current
2 2 2 2 2 2
is in same direction so they will attract each other.
Magnetic field is equal in magnitude at P so magnetic B = 0 as current due to rotating charge is zero.
field at P is zero.
(B) µ = 0 as current due to rotating charge is zero.

Q
B B +• •+


P
F a
b M
Magnetic field at P is in the same direction.
•–
–•
Wires will attract as the current is in the same direction.
(C) Magnetic field at P is in opposite direction due to

+
two wires and has same magnitude. So net magnetic P
field is zero at P. Wires will attract each other as current E=0
is in the same direction.
Electric field will cancel out due to symmetry
(D) Magnetic field will be in opposite direction and wires
Kq Kq
will repel each other as current is in opposite sense. V=– ×3+ ×3≠0
a b

Sol 37: A → p, r, s; B → r, s; C → p, q; D → r, s B is not zero as current due to rotating charge is non-


zero.
Electric field is zero at point M
P hysi cs | 21.91

µ = INA →
Sol 4: (A, B, D) If both E and B are zero, then Fe and
as I ≠ 0 ⇒ µ ≠ 0 →
Fm both are zero. Hence, velocity may remain constant.
– + – Therefore, option (a) is correct.
If E = 0, B ≠ 0 but velocity is parallel or antiparallel
→ →
to magnetic field, then also Fe and Fm both are zero.
P Q Hence, option (b) is also correct.
→ →
If E ≠ 0, B ≠ 0 but Fe + Fm = 0, then also velocity may
– –
+ remain constant or option (d) is also correct.
Electric field is zero.By symmetry electric field will
cancel out each other. Sol 5: (A, B, D) Magnetic force does not do work. From
−Kq Kq work-energy theorem:
×4+ ×2 ≠ 0
V =  5a a  1
  WFe = ∆KE or (qE)(2a) = m[4v2 – v2]
 2  2 2
 
3  mv 2 
Let I be the current due to moving charge or E =  
4  qa 
2xµ0 Ia2  
µ0 I ≠0
So B = – 3 At P, rate of work done by electric field
2a 2 ( )
2a2 2 → →

µ = INA = F e . v = (qE)(v) cos 0°


µ = 2× Ia2 – Ia2= Ia2  3 mv 2  3  mv 3 
= q  v=  
 4 qa  4  a 
 

Previous Years’ Questions Therefore, option (b) is also correct. Rate of work done
→ →
at Q: of electric field = F e . v = (qE)(2v)cos 90° = 0 and
of magnetic field is always zero. Therefore, option (d) is
Sol 1: (C) cφ = BINA
also correct.
 BNA  →
∴ φ=  I Note that F e = qE î
 c 

mv P 2km
Sol 2: (C) If B2> B1, critical temperature, (at which Sol 6: (A, C) r = = =
resistance of semiconductors abruptly becomes zero) Bq Bq Bq
in case 2 will be less than compared to case1. m
i.e., r ∝
Using iron core, value of magnetic field increases. So, q
deflection increases for same current. Hence, sensitivity
If K and B are same.
increases.
Soft iron can be easily magnetized or demagnetized. 1 4 16
i.e., r :r :r = : : =1:2:3
H+ He+ O2 + 1 1 2
Sol 3: (D) With increase in temperature, TC is decreasing. Therefore, He+ and O2+ will be deflected equally but H+
having the least radius will be deflected most.
TC(0) = 100 K
TC = 75 K at B = 7.5 T
Hence, at B = 5 T, TC should lie between 75 K and 100 K.
Hence, the correct option should be (b).
2 1 . 9 2 | Moving Charges and Magnetism


Sol 7: (A, C) F BA = 0, because magnetic lines are
X X X
parallel to this wire.
→ X X X
F CD = 0, because magnetic lines are antiparallel to this
wire. ×
→ → –
F CB is perpendicular to paper outwards and F AD is
perpendicular to paper inwards. These two forces
(although calculated by integration)cancel each other
but produce a torque which tend to rotate the loop in
clockwise direction about an axis OO’. Te Tp
∴ Te< Tp, te = and tp =
2 2
BqI
Sol 8: (A, C, D) v =
m or te< tp
→ →
v ⊥ B in region II. Therefore, path of particle is circle
in region II. Sol 10: (C, D) Y
E0, B0
v

X X X

X X
X X
If θ =0 or 10o
X X then particle moves in helical path with increasing pitch
along Y-axis.
If θ =90o then magnetic force on the particle is
Particle enters in region III if, radius of circular path, r >l zero and particle moves along Y-axis with constant
acceleration.
mv
or >l
Bq
Sol 11: (5)
BqI
or v>
m a
a/2
BqI mv P O
If v = ,r= = I, particle will turn back and path
m Bq
length will be maximum. If particle returns to region I,

time spent in region II will be:


I = J × πa2
T πm
t= = , which is independent of v. µ0 J × πa2 µ0 J × π a2
2 Bq B= − ×
2πa 3a 4
2π ×
mv 2
Sol 9: (B, D) r = or r ∝ m
Bq
1 1 
∴ re< rp as me< mp ⇒B=
µ0 Ja  − 
 2 12 

2πm 5
Further, T = or T ∝ m ⇒B=
µ0 Ja ×
Bq 12
P hysi cs | 21.93

Sol 12: (B) M = I × Area of loop k̂ Sol 16: (3) Case – I

 πa2  π  Case-I Case-II


=I × a2 × × 4  kˆ =I × a2  + 1 kˆ
 4×2  2 

Sol 13: (D) I I I I


X0 X0
R/2
R
r< ; B=
0
2 P P
X0/3 X0/3
R
B at r = 1  µ0   3I 
2 B1 =   
2  2π 
 x0 
µ0 JR µ0 JR
⇒B= − =0 mv
2×2 2×2 R1 =
qB1
R
B at r > Case - II
2
Case-I Case-II
µ JR µ J × π R 2
⇒B=0 − 0 ×
2 2πr 4

µ0L  R 2 
=B r −  I I I I
2  4r  X0 X0

R
If we put r = ,B=0
2 P P
X0/3 X0/3
∴ B is continuous at r = R/2
mv
R1 =
Sol 14: (A, C) So magnetic field is along –ve, z-direction. qB2
πM
Time taken in the magnetic field =10 × 10−1 = R1 B2 1/3
6QB ⇒ = = =3
R2 B1 1/9
πM 1000πM 50πM
=B = =
−3
60 × 10 Q 60Q 3Q Sol 17: (C) The net magnetic field at the given point
will be zero if.
y  


 2 ( 3i + j ( | Bwires |=| Bloop |


o
30
o 30
µ0 I a µ0 Ia2
⇒ 2 × =
2 2 3/2
2π a2 + h2 a2 + h2 2(a + h )

x ⇒ h ≈ 1.2a


4i

Sol 15: (B) The direction of magnetic field at the given point due
to the loop is normally out of the plane. Therefore, the
−2GMm 1 GM net magnetic field due the both wires should be into
+ mv 2 =
0⇒v=
2 the plane. For this current in wire I should be along PQ
L 2 L
and that in wire RS should be along SR.
Note: The energy of mass ‘m’ means its kinetic energy
(KE) only and not the potential energy of interaction Sol 18: (B)
between m and the two bodies (of mass M each) – µ0 I µ0 I2a2
τ = MB sin θ = Iπa2 × 2 × sin 30o =
which is the potential energy of the system. 2πd 2d
2 1 . 9 4 | Moving Charges and Magnetism


Sol 19: (A, B, C) F = 2I(L + R)[iˆ × B]
ˆ

.
2(L+R)

Sol 20: (A, D) I1 = I2

⇒ neA1 v1 =
neA2 v 2
⇒ d1 w1 v1 =
d2 w2 v 2

Now, potential difference developed across MK


V = Bvw
V1 v1 w1 d2
⇒ = =
V2 v 2 w2 d1

Sol 21: (A, C) As I1 = I2

n1 w1d1 v1 = n2 w2d2 v 2

V2 B2 v 2 w2  B2 w2  n1 w1d1  B2n1
Now
= =   =
 
V1 B2 v1 w1  B1 w1  n2 w2d2  B1n2

Вам также может понравиться